page | 70barium swallow - 1 file download

54
MyPasTest: MRCS A Online - Jan Exam 2015 02. Anatomy; Thorax(120Qs) ----------------------------------------------------------------------------------------------------------------- ------------------------------------------------------------------------------------------------------------------------------------------------------------------------- Dr Mohammed Shamsul Islam Khan, Medical Officer, Clinical Neuro-Surgery, National Institute of Neuro-Sciences and Hospital Sher-E-Bangla Nagar, Dhaka-1207, Bangladesh. Mobile: +880 1713 455 662, E-mail: [email protected] Page | 70 01. A 17-year-old male is stabbed over the left lower ribs whilst travelling home from a night out. He is alert and haemodynamically stable on admission. A naso gastric tube is inserted and a chest x- ray shows this to be in the thoracic cavity and a traumatic diaphragmatic injury is thought likely. What is the best imaging technique to visualise the anatomy of the diaphragm? Select one answer only. Abdominal CT« YOUR ANSWER Barium swallow Chest CT Gastrograffin swallow MRI« CORRECT ANSWER. Diaphragmatic injuries result from either blunt or penetrating trauma. A traumatic diaphragmatic rupture is more commonly diagnosed on the left side, perhaps because the liver obliterates the defect or protects it on the right side. In addition, the appearance of bowel, stomach or a nasogastric (NG) tube is more easily detected in the left side of the chest. Right diaphragmatic ruptures are rarely diagnosed in the early post-injury period. The liver often prevents herniation of other abdominal organs into the chest. This, however, may not be representative of the true incidence of laterality and autopsy studies have revealed that left- and right-sided ruptures occur almost equally. Blunt trauma produces large radial tears measuring 515 cm, most often at the posterolateral aspect of the diaphragm. In contrast, penetrating trauma usually create only small linear incisions or perforations, which are less than 2 cm in size and may often take some time, even years, to develop into diaphragmatic hernias. If a laceration of the left diaphragm is suspected, a NG tube should be inserted. If the tube appears in the thoracic cavity on the chest film, the need for special contrast studies can be eliminated. Minimally invasive endoscopic procedures (thoracoscopy) may be helpful in evaluating the injury to the diaphragm in indeterminate cases. Abdominal computed tomography scan is usually not helpful because of its poor visualisation of the diaphragm. Magnetic resonance imaging is more accurate in visualising the anatomy of the diaphragm. It is very sensitive and specific and so is the investigation of choice. Surgical repair is necessary, even for small tears, because the defect will not heal spontaneously. 02. A 32-year-old female complains of a long history of pain and paraesthesiae along the ulnar border of her left arm and forearm precipitated by placing her upper limb in certain positions. She reports episodes of dropping items in her left hand recently and also episodes of left arm swelling. On examination wasting of the small muscles of the left hand is noted. Which of the following is the most likely diagnosis? Select one answer only. Carpal tunnel syndrome« YOUR ANSWER Cubital tunnel syndrome Erbs palsy Klumpke’s paralysis

Upload: others

Post on 12-Dec-2021

3 views

Category:

Documents


0 download

TRANSCRIPT

MyPasTest: MRCS A Online - Jan Exam 2015 02. Anatomy; Thorax(120Qs)

-----------------------------------------------------------------------------------------------------------------

------------------------------------------------------------------------------------------------------------------------------------------------------------------------- Dr Mohammed Shamsul Islam Khan, Medical Officer, Clinical Neuro-Surgery, National Institute of Neuro-Sciences and Hospital Sher-E-Bangla

Nagar, Dhaka-1207, Bangladesh. Mobile: +880 1713 455 662, E-mail: [email protected]

Page |

70

01. A 17-year-old male is stabbed over the left lower ribs whilst travelling home from a night out. He is alert and haemodynamically stable on admission. A naso gastric tube is inserted and a chest x-ray shows this to be in the thoracic cavity and a traumatic diaphragmatic injury is thought likely. What is the best imaging technique to visualise the anatomy of the diaphragm? Select one answer only.

Abdominal CT« YOUR ANSWER

Barium swallow

Chest CT

Gastrograffin swallow

MRI« CORRECT ANSWER. Diaphragmatic injuries result from either blunt or penetrating trauma. A traumatic diaphragmatic rupture is more commonly diagnosed on the left side, perhaps because the liver obliterates the defect or protects it on the right side. In addition, the appearance of bowel, stomach or a nasogastric (NG) tube is more easily detected in the left side of the chest. Right diaphragmatic ruptures are rarely diagnosed in the early post-injury period. The liver often prevents herniation of other abdominal organs into the chest. This, however, may not be representative of the true incidence of laterality and autopsy studies have revealed that left- and right-sided ruptures occur almost equally. Blunt trauma produces large radial tears measuring 5–15 cm, most often at the posterolateral aspect of the diaphragm. In contrast, penetrating trauma usually create only small linear incisions or perforations, which are less than 2 cm in size and may often take some time, even years, to develop into diaphragmatic hernias. If a laceration of the left diaphragm is suspected, a NG tube should be inserted. If the tube appears in the thoracic cavity on the chest film, the need for special contrast studies can be eliminated. Minimally invasive endoscopic procedures (thoracoscopy) may be helpful in evaluating the injury to the diaphragm in indeterminate cases. Abdominal computed tomography scan is usually not helpful because of its poor visualisation of the diaphragm. Magnetic resonance imaging is more accurate in visualising the anatomy of the diaphragm. It is very sensitive and specific and so is the investigation of choice. Surgical repair is necessary, even for small tears, because the defect will not heal spontaneously. 02. A 32-year-old female complains of a long history of pain and paraesthesiae along the ulnar border of her left arm and forearm precipitated by placing her upper limb in certain positions. She reports episodes of dropping items in her left hand recently and also episodes of left arm swelling. On examination wasting of the small muscles of the left hand is noted. Which of the following is the most likely diagnosis? Select one answer only.

Carpal tunnel syndrome« YOUR ANSWER

Cubital tunnel syndrome

Erbs palsy

Klumpke’s paralysis

MyPasTest: MRCS A Online - Jan Exam 2015 02. Anatomy; Thorax(120Qs)

-----------------------------------------------------------------------------------------------------------------

------------------------------------------------------------------------------------------------------------------------------------------------------------------------- Dr Mohammed Shamsul Islam Khan, Medical Officer, Clinical Neuro-Surgery, National Institute of Neuro-Sciences and Hospital Sher-E-Bangla

Nagar, Dhaka-1207, Bangladesh. Mobile: +880 1713 455 662, E-mail: [email protected]

Page |

71

Thoracic outlet syndrome« CORRECT ANSWER. Cervical rib is present in about 0.5% of the population, of which 60% are symptomatic. Symptoms due to the presence of a cervical rib depend on the structure it is compressing. Neurological symptoms are the most common presentation, usually compression of the C8 and T1 nerve roots, which causes pain and paraesthesiae on the ulnar aspect of the arm and forearm and wasting of the small muscles of the hand. Vascular changes are seen less often. The arm can become swollen as a result of venous compression. Compression of the subclavian artery can lead to thrombus formation, emboli, ischaemic changes, and even gangrene.

03. A 65-year-old diabetic with hypertension and a long smoking history is due to undergo a CABG for triple vessel disease. Which of the following grafts would be likely to have the highest patency rates at 10 years? Select one answer only.

Left internal mammary« YOUR ANSWER

Long saphenous vein

Radial artery

PTFE graft

Cryopreserved allograft vein. During the first year following a CABG up to 15% of vein grafts occlude. Between 1–6 years the graft attrition is 1% to 2% per year and between 6–10 years it is 4% per year. By 10 years after bypass surgery, only 60% are patent and only 50% are free of significant stenosis. The internal thoracic (mammary) artery has a reported patency of up to 90% at 10 years. The left internal thoracic artery is usually anastomosed to the left anterior descending artery (or anterior interventricular artery). A number of studies have been carried out to assess the use of alternative grafts for coronary bypass surgery, including PTFE and cryopreserved allograft veins (CAVs). The results so far have been disappointing. 04. A 32-year-old man is brought in after a RTA in which he was doing 40 mph and collided with an on-coming vehicle after he lost concentration momentarily. He was not wearing a seatbelt and a sternal fracture is seen on imaging. There is concern about a myocardial contusion. What is the best way to establish this diagnosis? Select one answer only.

ECG« YOUR ANSWER

Direct inspection of myocardium

Serial troponin levels and ECG« CORRECT ANSWER

Transoesophageal echo

Transthoracic echo. Blunt cardiac injury, commonly seen in patients with decelerating trauma, can cause valvular disruption, pericardial effusion, myocardial muscle contusion and cardiac chamber rupture. Patients with myocardial contusion may complain of chest discomfort/pain and may be hypotensive. However, cardiogenic shock is rarely seen with myocardial contusion alone.

MyPasTest: MRCS A Online - Jan Exam 2015 02. Anatomy; Thorax(120Qs)

-----------------------------------------------------------------------------------------------------------------

------------------------------------------------------------------------------------------------------------------------------------------------------------------------- Dr Mohammed Shamsul Islam Khan, Medical Officer, Clinical Neuro-Surgery, National Institute of Neuro-Sciences and Hospital Sher-E-Bangla

Nagar, Dhaka-1207, Bangladesh. Mobile: +880 1713 455 662, E-mail: [email protected]

Page |

72

A two-dimensional echocardiography may reveal an abnormal ventricular wall motion. The electrocardiographic changes are variable: They include multiple premature ventricular contractions, unexplained sinus tachycardia, atrial fibrillation, bundle branch block (usually right), non-specific ST and T wave changes and features suggestive of frank myocardial infarction. In current practice, the widespread availability of cardiospecific troponin I and troponin T assays has made it easier to detect myocardial injury, especially in patients with no major signs of cardiac injury. Troponin I and troponin T have also facilitated the stratification of patients at risk for life threatening complications. Serial measurements of troponin I or T in combination with ECG have shown to be sufficient for identifying the vast majority of patients at risk, thus avoiding extensive diagnostic screening. Moreover, patients with normal troponin I or T concentrations and ECG may be safely discharged after a period of observation. Ref: http://www.ncbi.nlm.nih.gov/pmc/articles/PMC1767619/ 05. A 64-year-old man is admitted with a tearing chest pain radiating through to his back. He is

haemodynamically stable and a CT angiogram shows a type A thoracic aneurysm with dissection.

Which of the following vessels normally arise from the aortic arch? Select one answer only.

Left subclavian artery« YOUR ANSWER (Correct)

Left vertebral artery

Right internal mammary artery

Right common carotid artery

Right subclavian artery. The arch of the aorta commences from the manubriosternal joint and passes backwards over the left bronchus to reach the body of T4 vertebra just to the left of the midline. The arch is crossed on its left side by the phrenic and vagus nerves as they pass downwards in front of and behind the lung root, respectively. The left vertebral artery and the right internal mammary artery come off the left and right subclavian arteries, respectively.

06. Following excision of a left cervical rib a 28-year-old female is found to have a milky, white fluid in her drain. What structure is most likely to have been damaged? Select one answer only.

Cisterna chyli« YOUR ANSWER

Hemiazygos vein

Left subclavican artery

Left subclavican vein

Thoracic duct« CORRECT ANSWER.

MyPasTest: MRCS A Online - Jan Exam 2015 02. Anatomy; Thorax(120Qs)

-----------------------------------------------------------------------------------------------------------------

------------------------------------------------------------------------------------------------------------------------------------------------------------------------- Dr Mohammed Shamsul Islam Khan, Medical Officer, Clinical Neuro-Surgery, National Institute of Neuro-Sciences and Hospital Sher-E-Bangla

Nagar, Dhaka-1207, Bangladesh. Mobile: +880 1713 455 662, E-mail: [email protected]

Page |

73

The thoracic duct leaves the cysterna chyli at the level of L1–L2, ascends into the thorax to the right of the descending thoracic aorta, crosses the midline gradually to reach the left border of the oesophagus (plane of Louis, T4), where it continues to run upwards, reaching the root of the neck. It then curves behind the carotid sheath and arches over the left subclavian artery to drain into the left brachiocephalic vein, although it can also drain into the other adjacent major veins. It carries lymph from the lower limbs, abdominal and pelvic regions, left thorax, left head and neck plus the left arm. 07. A 27-year-old male is rushed into A&E after being involved in a RTA. From the history he was a front seat passenger in a stolen car that crashed into a wall at approximately 70mph. He is complaining of generalised pain and on examination prominent marks are seen corresponding to his seatbelt. His trauma series chest x-ray shows an obliterated aortic knuckle and there is concern about blunt aortic rupture. Which of the following imaging modalities is the gold standard in diagnosing this? Select one answer only.

Aortography« YOUR ANSWER (Correct)

Contrast- enhanced CT of the chest

MRI

Transoesophageal echocardiography

Transthoracic echocardiography. Traumatic aortic disruption, a time-sensitive injury, is a common cause of sudden death after an automobile collision or a fall from great height. A complete tear through the tunica intima, media and adventitia usually leads to rapid exsanguination and death. In aortic rupture survivors, immediate death is prevented due to the vascular continuity maintained by a pseudoaneurysm within an intact adventitial layer or a mediastinal haematoma. A large mediastinal haematoma may shift the trachea to the right. This condition has a variable course ranging from a relatively clinically silent period due to the contained rupture (pseudoaneurysm), to rupture of the pseudoaneurysm, exsanguination and death. Radiographic findings may include a widened mediastinum, obliteration of the aortic knuckle, deviation of the trachea to the right, obliteration of the space between the pulmonary artery and the aorta (obscuration of AP (aorto-pulmonary) window), depression of the left main stem bronchus, deviation of the oesophagus (nasogastric tube) and fractures of the first or second rib or scapula. False-positive and false-negative findings occur with each radiographic sign and, rarely (1–2%), no mediastinal or initial chest X-ray abnormality is present in patients with great vessel injury. Although transoesophageal echocardiography is a useful, the less invasive diagnostic tool, aortography is the gold standard in the diagnosis of blunt aortic rupture. Helical contrast-enhanced computed tomography (CT) of the chest is also an accurate screening method for patients with suspected blunt aortic injury. However, a patient who is haemodynamically abnormal should not be placed in a CT scanner. In stable patients, if enhanced helical CT of the chest is negative for mediastinal haematoma and aortic rupture, no further diagnostic imaging is necessary. If it is positive for blunt aortic rupture, the extent of the injury can best be ascertained by aortography.

MyPasTest: MRCS A Online - Jan Exam 2015 02. Anatomy; Thorax(120Qs)

-----------------------------------------------------------------------------------------------------------------

------------------------------------------------------------------------------------------------------------------------------------------------------------------------- Dr Mohammed Shamsul Islam Khan, Medical Officer, Clinical Neuro-Surgery, National Institute of Neuro-Sciences and Hospital Sher-E-Bangla

Nagar, Dhaka-1207, Bangladesh. Mobile: +880 1713 455 662, E-mail: [email protected]

Page |

74

08. In a 27-year-old restrained passenger from a high speed collision with a tree, you suspect a deceleration injury to the aorta. Which feature on chest X-ray would most reliably support your suspicion? Single best answer question – choose ONE true option only.

Obliteration of the aortic knuckle« YOUR ANSWER

Obscuration of the aorto-pulmonary window

Presence of a pleural cap

Depression of the left main stem bronchus

Widened mediastinum« CORRECT ANSWER. All of the above can be associated with a traumatic aortic injury. However, they are non-specific signs with high false positive and false negative rates. Occasionally, no chest X-ray changes are visible following traumatic aortic injury. In a trauma supine CXR, a widened mediastinum is the most consistent feature of aortic disruption. About 3% of these patients would prove to have aortic injury on contrast CT or aortic angiogram. 09. You are called to the Trauma Room of the Emergency Department and find a 34-year-old motorcyclist with a blunt chest injury. On examination you notice reduced breath sounds and a dull percussion note over the right hemithorax. Trauma care is initiated and includes the insertion of an intercostal drain which quickly yields 1600mls of blood. His blood pressure is recorded as 100/65mmHg. What is the most appropriate management of this injury?

Continued non-surgical management« YOUR ANSWER

Emergency thoracoscopy

Emergency thoracotomy in the Emergency Department

Emergency thoracotomy in the Operating Theatre« CORRECT ANSWER

Massive blood transfusion. This patient has a massive haemothorax, which is defined as the rapid accumulation of more than 1500mL (or one third) of a patient‟s blood volume in the chest cavity. ATLS guidelines recommend that emergency thoracotomy is performed by a thoracic surgeon in an operating theatre environment in such cases. Blood transfusion may be required, but massive transfusion may not be necessary if surgical methods of haemostasis are rapidly delivered. Emergency room thoracotomy should only be performed in the context of: i) penetrating chest injury plus cardiac arrest (with previously witnessed cardiac activity) or unresponsive hypotension (<70mmHg SBP). ii) blunt chest injury with massive haemothorax and unresponsive hypotension (<70mmHg). 10. During pericardiectomy, sudden bleeding was noticed due to accidental injury to a major vascular structure in the pericardium. The surgeon inserted his left index finger through the transverse pericardial sinus, pulled forward on the two large vessels lying ventral to his finger and compressed these vessels with his thumb to control bleeding. Which vessels were these? Single best answer question – choose ONE true option only.

MyPasTest: MRCS A Online - Jan Exam 2015 02. Anatomy; Thorax(120Qs)

-----------------------------------------------------------------------------------------------------------------

------------------------------------------------------------------------------------------------------------------------------------------------------------------------- Dr Mohammed Shamsul Islam Khan, Medical Officer, Clinical Neuro-Surgery, National Institute of Neuro-Sciences and Hospital Sher-E-Bangla

Nagar, Dhaka-1207, Bangladesh. Mobile: +880 1713 455 662, E-mail: [email protected]

Page |

75

Pulmonary trunk and brachiocephalic trunk« YOUR ANSWER

Pulmonary trunk and aorta« CORRECT ANSWER

Pulmonary trunk and superior vena cava

Superior vena cava and aorta

Superior vena cava and right pulmonary artery. The transverse pericardial sinus is an area of the pericardial cavity located behind the aorta and pulmonary trunk and anterior to the superior vena cava. Therefore, the two large vessels lying ventral to his finger are the pulmonary trunk and aorta; the large vessel lying dorsal to his finger is the superior vena cava. 11. A 58-year-old diabetic male presents with chest pain radiating to the back and an initial chest X ray shows a widened mediastinum. Further imaging demonstrates a thoracic aortic aneurysm and the decision is made for surgical management. What type of incision allows the best access to the thoracic aorta? Select one answer only.

Median sternotomy« YOUR ANSWER

Rooftop incision

Thoracoabdominal

Thoracoscopic

Thoracotomy« CORRECT ANSWER. Thoracic aortic aneurysms are far less common than abdominal aneurysms. Previous dissection may lead to weakness of the aortic wall, and thus predispose to aneurysm formation. There is no common association with neurofibromatosis. They are just as likely to rupture as abdominal aneurysms; according to the law of LaPlace the wall tension is proportional to the radius at a given pressure. Access to the thoracic aorta is best afforded by a thoracotomy rather than a median sternotomy, as it is a posterior mediastinal structure.

12. A 38-year-old man is diagnosed with a sliding hiatus hernia. Which pair of structures travel through the same diaphragmatic aperture as the hernia? Select one answer only.

Azygous vein and inferior vena cava« YOUR ANSWER

Azygous vein and right phrenic nerve

Hemiazygous vein and oesophagus

Vagus nerve and azygous vein

Vagus nerve and oesophagus« CORRECT ANSWER. The sliding hernia occurs as a result of the stomach and oesophagus moving cranially through the oesophageal opening of the diaphragm into the thoracic cavity.

MyPasTest: MRCS A Online - Jan Exam 2015 02. Anatomy; Thorax(120Qs)

-----------------------------------------------------------------------------------------------------------------

------------------------------------------------------------------------------------------------------------------------------------------------------------------------- Dr Mohammed Shamsul Islam Khan, Medical Officer, Clinical Neuro-Surgery, National Institute of Neuro-Sciences and Hospital Sher-E-Bangla

Nagar, Dhaka-1207, Bangladesh. Mobile: +880 1713 455 662, E-mail: [email protected]

Page |

76

Three sets of structures pierce the diaphragm at three vertebral levels:

1. T8 = Right phrenic nerve and IVC; these pierce the diaphragm through the central tendon (caval opening)

2. T10 = Vagus nerve and Oesophagus; these pierce the diaphragm through its right crus (oesophageal opening)

3. T12 = Azygous vein, thoracic duct and abdominal aorta; these pass posterior to the diaphragm through the aortic opening.

13. A 20-year-old man presents to A&E with a traumatic pneumothorax. He is obviously tachypnoeic but maintains adequate oxygen saturation. Which of the following is least likely to contribute to his respiratory effort? Select one answer only.

Contraction of the intercostal muscles« YOUR ANSWER

Descent of the hemidiaphragms

An increase in the vertical dimension of the chest

The long thoracic nerve of Bell (supplying the serratus anterior)

Upward/forward movement of the first rib« CORRECT ANSWER. In cases of impaired ventilation, tachypnoea can compensate the impaired gas exchange to an extent until fatigue of the respiratory muscles occurs. A number of factors are involved in respiration: - An increase in the vertical dimension of the chest on inspiration - An upwards and outwards movement of the ribs - A rise and fall of the hemi diaphragms. The serratus anterior (supplied by the long thoracic nerve) is also involved in respiration. Note that the first rib does not move during respiration. 14. A 76-year-old male is undergoing an oesophagectomy for adenocarcinoma of the distal oesophagus thought to be secondary to long standing GORD. Which part of the diaphragm does the oesophagus pass through? Select one answer only.

Central tendon« YOUR ANSWER

Left crus

Median arcuate ligament

Peripheral muscular part

Right crus« CORRECT ANSWER. It is important to know what structures pass through the diaphragm, at what vertebral levels, and also which specific part of the diaphragm. The aortic opening lies behind the diaphragm at T12 at vertebral level T12. It contains the aorta, thoracic duct, and often the azygos vein. The oesophageal hiatus is at vertebral level T10 behind the seventh costal cartilage, and is formed

MyPasTest: MRCS A Online - Jan Exam 2015 02. Anatomy; Thorax(120Qs)

-----------------------------------------------------------------------------------------------------------------

------------------------------------------------------------------------------------------------------------------------------------------------------------------------- Dr Mohammed Shamsul Islam Khan, Medical Officer, Clinical Neuro-Surgery, National Institute of Neuro-Sciences and Hospital Sher-E-Bangla

Nagar, Dhaka-1207, Bangladesh. Mobile: +880 1713 455 662, E-mail: [email protected]

Page |

77

by the medial fibres of the right crus of the diaphragm as they arch across to the left side of the midline. They form a tubular envelope around the oesophagus with an elliptical opening. In addition to the oesophagus, it transmits the vagus nerve, left gastric artery and vein. The inferior vena cava passes through at T8 in the central tendon. The right phrenic nerve passes with it. The sympathetic trunk passes posterior to the medial arcuate ligament lying on psoas major. Each half of the diaphragm is supplied by its own phrenic nerve (C3, C4, C5) which is both motor and sensory. The intercostal nerves send some proprioceptive fibres to the periphery of the diaphragm. The greater, lesser and least splanchnic nerves pierce each crus. 15. A 4-year-old boy is rushed into A&E by his mum who is concerned he has aspirated on one of his lego bricks he likes to put into his mouth. On examination, the child appears well with sats of 98%. A chest X-ray shows a foreign body impacted in the right main bronchus. Which of the following is the most appropriate management? Select one answer only.

Conservative management« YOUR ANSWER

CT guided removal

Fibre-optic bronchoscopy to attempt to remove it

Rigid bronchoscopy to attempt to remove it« CORRECT ANSWER

Thoracoscopic removal. Inhaled or aspirated foreign bodies are more commonly seen in children than adults. A history of foreign body aspiration must be taken seriously, as the consequences of a retained foreign body in the lung may be significant. Following a careful history and examination, posteroanterior and lateral chest radiography may help to localise the foreign object. However, some objects may be radiolucent and may not be seen on the chest radiograph. Further investigation is imperative, the next step being either fibre-optic or preferably rigid bronchoscopy. Rigid bronchoscopy is preferred for recovery of foreign body aspiration because it allows protection of the airway and controlling the foreign body during recovery. The anatomy of the bronchial tree, in particular a less angulated right main bronchus, makes this the commonest site of impaction. 16. A 62-year-old male with alcoholic liver disease is deteriorating in ICU following insertion of a Sengstaken-Blakemore tube for bleeding oesophageal varices. His chest X-ray shows a pneumomediastinum and he is thought to have perforated his oesophagus. Which of the following structures which run adjacent to the oesophagus makes an indentation on the left lung, but not the right lung? Select one answer only.

Azygos vein« YOUR ANSWER

Oesophagus

Phrenic nerve

Superior vena cava

MyPasTest: MRCS A Online - Jan Exam 2015 02. Anatomy; Thorax(120Qs)

-----------------------------------------------------------------------------------------------------------------

------------------------------------------------------------------------------------------------------------------------------------------------------------------------- Dr Mohammed Shamsul Islam Khan, Medical Officer, Clinical Neuro-Surgery, National Institute of Neuro-Sciences and Hospital Sher-E-Bangla

Nagar, Dhaka-1207, Bangladesh. Mobile: +880 1713 455 662, E-mail: [email protected]

Page |

78

Thoracic duct« CORRECT ANSWER. Impressions on the mediastinal surface of the right lung include the trachea, vagus, superior vena cava, right atrium and subclavian artery. The oesophagus grooves the left lung above the arch of the aorta and below the hilum. There is also a vertical groove for the oesophagus on the right lung. 17. During a coronary angiography a tight stenosis is noted at the proximal aspect of the left coronary artery. Which of the following is the most important branch of the left coronary artery? Select one answer only.

Anterior interventricular« YOUR ANSWER (Correct)

Atrioventricular nodal

Circumflex

Marginal

Posterior interventricular. The left coronary artery divides into the left anterior interventricular artery also known as the left anterior descending artery (LAD) and circumflex artery. It‟s most important branch is the anterior interventricular artery which supplies the anterior aspect of both ventricles and passes around the apex of the heart to anastomose with the posterior interventricular artery. The right coronary artery gives off a posterior interventricular branch. The right coronary artery supplies the AV node in 80% of cases.

18. A 25-year-old female presents with pain in the right hand which is associated with a tingling sensation and weakness. After further investigation she is diagnosed with thoracic outlet syndrome. The affected neurovascular bundle runs between scalenus anterior and medius. Where do these muscles insert?

Body of the sternum« YOUR ANSWER

Clavicle

First rib« CORRECT ANSWER

Manubrium

Second rib. Thoracic outlet syndrome is caused by compression of the neurovascular bundle that passes between the anterior and middle scalene muscles. The brachial plexus and the subclavian artery and (rarely) the subclavian vein can be affected, leading to a variety of upper limb, neck and upper back symptoms. Cervical ribs, Pancoast‟s tumour and preganancy can all cause the condition, but it is usually secondary to trauma or repetitive strain injuries. The scalene muscle is composed of three pairs of constituent muscles, namely scalenus anterior (originating at anterior tubercles of the transverse processes of C3-6 vertebrae), scalenus medius (originating from the posterior tubercles of C2-6 vertebrae) and scalenus posterior (originating from

MyPasTest: MRCS A Online - Jan Exam 2015 02. Anatomy; Thorax(120Qs)

-----------------------------------------------------------------------------------------------------------------

------------------------------------------------------------------------------------------------------------------------------------------------------------------------- Dr Mohammed Shamsul Islam Khan, Medical Officer, Clinical Neuro-Surgery, National Institute of Neuro-Sciences and Hospital Sher-E-Bangla

Nagar, Dhaka-1207, Bangladesh. Mobile: +880 1713 455 662, E-mail: [email protected]

Page |

79

the posterior tubercles of the transverse processes of C4-6). The anterior and medius muscles insert into the first rib, the posterior component inserts into the second. 19. A 62-year-old ex-smoker has been diagnosed with non-small cell lung carcinoma by endobronchial biopsy. At standard mediastinoscopy, what is the least likely lymph node station to be sampled? Single best answer question – choose ONE true option only.

Paratracheal nodes« YOUR ANSWER

Subcarinal nodes

Tracheobronchial nodes

Aortopulmonary nodes« CORRECT ANSWER

Pretracheal nodes . During mediastinoscopy, the pretracheal fascia is incised and blunt dissection creates a tunnel inferiorly. The linear passage of the mediastinoscope along this tunnel allows visualisation of the nodes lying to the front (pretracheal) and sides (paratracheal) of the trachea. The subcarinal and tracheobronchial nodes can also be reached at the distal end of this tunnel. The aortopulmonary nodes, however, cannot easily be reached as the aorta is “in the way” of the advancing finger or mediastinoscope.

20. Following a difficult tracheostomy, the surgeon is concerned about the nerve supply to the trachea being damaged. Which of the following nerves supply the trachea? Select one answer only.

Phrenic nerve« YOUR ANSWER

Glossopharyngeal nerve

Recurrent laryngeal nerve« CORRECT ANSWER

Superior laryngeal nerve

External laryngeal nerve. The trachea attaches to the larynx via the cricotracheal membrane at about the level of the sixth cervical vertebrae, and descends through the neck and superior mediastinum to terminate at about the level of the disc between the fourth and fifth thoracic vertebrae, although this bifurcation descends as the trachea is stretched during inspiration. The tracheal rings consist of hyaline cartilage. It is supplied by the recurrent laryngeal nerve which is sensory below the level of the vocal cords and motor to all muscles of the larynx with the exception of the cricothyroid, which is supplied by superior laryneal nerve. 21. A 76-year-old male who is a long term smoker presents with a headache and facial swelling. On further questioning he reports episodes of haemoptysis and 11/2 stone in weight loss. Examination reveals a plethoric face and neck, and distended veins on his upper chest and neck. A CT scan reveals a bronchial tumour causing SVC obstruction. At what level does the SVC enter the right atrium? Select one answer only.

At the level the azygos vein drains into it« YOUR ANSWER

MyPasTest: MRCS A Online - Jan Exam 2015 02. Anatomy; Thorax(120Qs)

-----------------------------------------------------------------------------------------------------------------

------------------------------------------------------------------------------------------------------------------------------------------------------------------------- Dr Mohammed Shamsul Islam Khan, Medical Officer, Clinical Neuro-Surgery, National Institute of Neuro-Sciences and Hospital Sher-E-Bangla

Nagar, Dhaka-1207, Bangladesh. Mobile: +880 1713 455 662, E-mail: [email protected]

Page |

80

Behind the first costal cartilage

Behind the third costal cartilage« CORRECT ANSWER

Level of sternal angle

Vertebral level T8. The SVC drains all the structures above the diaphragm except the heart and lungs. It also receives the azygos vein, which drains the lumbar and subcostal regions. The SVC is formed behind the first costal cartilage by the union of the right and left brachiocephalic veins. It ends behind the third costal cartilage as it enters the right atrium. The SVC has no valves. The thoracic duct drains into the left brachiocephalic vein (or sometimes into the subclavian or internal jugular vein). 22. A 17-year-old girl is brought to A&E following a horse riding accident. She is very distressed and in a lot of pain. From the history the horse fell backwards and landed on the left side of her chest. On examination there is extensive bruising over the left chest and it can be seen to move in when she inspires. Auscultation reveals bilateral air entry and her trachea is central. What is the most likely diagnosis? Select one answer only.

Chylothorax« YOUR ANSWER

Flail chest « CORRECT ANSWER

Haemothorax

Pneumothorax

Tension pneumothorax. A flail chest occurs when a segment of the thoracic wall does not have bony continuity with the rest of the thoracic cage. Flail chest classically results from blunt trauma, as in high-speed road-traffic accidents. A transfer of significant kinetic energy to the rib cage can cause fracture of the ribs in multiple areas leading to a segment of the thoracic wall to „float‟ independently of the rest of the chest wall. A flail chest can arise when two or more ribs are fractured in two or more places. The presence of a flail chest segment results in severe disruption of normal chest wall movement leading to paradoxical breathing (when the injured segment of the thoracic cage moves in an opposite direction to the rest of the chest wall). Initial management of flail chest includes adequate ventilation, administration of humidified oxygen, adequate analgesia and fluid resuscitation. However, in the absence of systemic hypotension (from other associated causes), intravenous fluid resuscitation should be carefully monitored to prevent over hydration. The injured lung in a flail chest is very sensitive to both under-resuscitation and fluid overload. The definitive management of flail chest is to re-expand the affected (contused or collapsed) lung, ensure adequate oxygenation, provide sufficient pain relief and judicious fluid resuscitation. Since prevention of hypoxia is of paramount importance some patients will benefit from a short period of intubation and ventilation, the timing guided by the respiratory rate, arterial oxygen tension and other vital respiratory parameters. Surgical stabilisation of the chest is an option but is rarely necessary in the management of flail chest.

MyPasTest: MRCS A Online - Jan Exam 2015 02. Anatomy; Thorax(120Qs)

-----------------------------------------------------------------------------------------------------------------

------------------------------------------------------------------------------------------------------------------------------------------------------------------------- Dr Mohammed Shamsul Islam Khan, Medical Officer, Clinical Neuro-Surgery, National Institute of Neuro-Sciences and Hospital Sher-E-Bangla

Nagar, Dhaka-1207, Bangladesh. Mobile: +880 1713 455 662, E-mail: [email protected]

Page |

81

23. You have performed a liver biopsy, and shortly after the procedure the patient develops pain on

the tip of his right shoulder. Which nerve is most likely to be responsible for his pain? Single best

answer - choose ONE true option only.

Right phrenic nerve« YOUR ANSWER (Correct)

Axillary nerve

Right vagus

Right sympatheticus

Intercostobrachial nerve.

The phrenic nerve on both sides originates from the ventral rami of the third to fifth cervical nerves. It passes inferiorly down the neck to the lateral border of the scalenus anterior, then it passes medially across the border of scalenus anterior parallel to the internal jugular vein that lies inferomedially.

The right phrenic nerve pierces the diaphragm in its tendinous portion just slightly lateral to the inferior vena caval foramen. It then forms three branches on the inferior surface of the diaphragm: anterior, lateral and posterior. These ramify out in a radial manner from the point of perforation to supply all but the periphery of the muscle.

24. You are required to insert a chest drain in a patient with penetrating trauma. Which anatomical landmark is least useful to you? Single best answer question – choose ONE true option only.

Anterior border of latissimus dorsi« YOUR ANSWER

The mid-clavicular line« CORRECT ANSWER

The sixth rib

The mid-axillary line

Inferolateral border of pectoralis major. The “safe triangle” for the insertion of an intercostal drain is bounded anteriorly by the inferolateral border of pectoralis major, posteriorly by the anterior border of latissimus dorsi and inferiorly by the axial plane at the level of the nipple. In practice, however, one must note that the position of the nipple is highly variable and so the drain is best sited within the 5th intercostal space (i.e. immediately above the 6th rib as located by palpation). The mid-axillary line may be used within this triangle to help guide placement of the incision. The mid-clavicular line should not be used for placement of an intercostal tube drain. 25. A pre-operative patient is on β-blockers. Where in the thoracic cage are the beta-1 adrenoceptors concentrated? Single best answer question – choose ONE true option only.

Cardiac ventricles« YOUR ANSWER (Correct)

Pulmonary trunk

Aortic arch

Lungs

Carotid sinus.

MyPasTest: MRCS A Online - Jan Exam 2015 02. Anatomy; Thorax(120Qs)

-----------------------------------------------------------------------------------------------------------------

------------------------------------------------------------------------------------------------------------------------------------------------------------------------- Dr Mohammed Shamsul Islam Khan, Medical Officer, Clinical Neuro-Surgery, National Institute of Neuro-Sciences and Hospital Sher-E-Bangla

Nagar, Dhaka-1207, Bangladesh. Mobile: +880 1713 455 662, E-mail: [email protected]

Page |

82

The beta-1 adrenoceptors are located within the ventricles of the heart. The atria contain cholinergic receptors. The aortic arch contains baroreceptors. The Lungs contain beta-2 adrenoceptors. The carotid sinus contains baroreceptors and is in the neck, not the thorax. 26. A 51-year-old female is rushed in following a road traffic accident. The patient was haemodynamically stable and a right sided haemothorax was noted. A CT scan showed a contained haematoma of the azygos vein. At what level does the azygos vein enter the superior vena cava? Select one answer only.

T2« YOUR ANSWER

T4« CORRECT ANSWER

T6

T8

T10. The azygos vein receives blood from the posterior intercostal veins and segmental vains of the abdomen, It is located on the right, and on the left a variable pattern exists, which mostly involves an accessory hemiazygos vein (superiorly) and azygos vain (inferiorly). The azygos vein is formed at the level of the right renal vein (either as a posterior tributary of the IVC, or as a confluence of the right ascending lumbar vein and right subcostal vein). It passes through the diaphragm via the aortic opening at T12 and ascends on the right side of the vertebral bodies posterior to the oesophagus. It terminates by arching over the hilum of the right lung to enter the SVC at T4 level. The azygos vein does not extend higher than T4, and therefore the 2nd, 3rd and 4th right intercostal veins drain into the right superior intercostal vein which itself drains into the azygos vein. Other tributaries draining into it include the lower 8 right posterior intercostal veins, bronchial and oesophageal veins and the 2 hemiazygos veins. 27. Following a very difficult right thoracic outlet syndrome decompression in a 35-year-old female there is some respiratory difficulty and there is concern the right phrenic nerve may be injured. Which of these descriptions most accurately describes the muscle relations of the phrenic nerve? Select one answer only.

Anterior to scalenus anterior« YOUR ANSWER (Correct)

Anterior to scalenusmedius

Anterior to trapezius

Posterior to scalenus anterior

Posterior to scalenusmedius. The phrenic nerve arises from C3, C4 and C5 deep to the scalenus anterior and medius muscles, and runs on scalenus anterior, over the anterior part of the dome of the pleura, to enter the mediastinum posterior to the subclavian vein. Here the right phrenic nerve spirals forward to lie on

MyPasTest: MRCS A Online - Jan Exam 2015 02. Anatomy; Thorax(120Qs)

-----------------------------------------------------------------------------------------------------------------

------------------------------------------------------------------------------------------------------------------------------------------------------------------------- Dr Mohammed Shamsul Islam Khan, Medical Officer, Clinical Neuro-Surgery, National Institute of Neuro-Sciences and Hospital Sher-E-Bangla

Nagar, Dhaka-1207, Bangladesh. Mobile: +880 1713 455 662, E-mail: [email protected]

Page |

83

the SVC, right atrium and inferior vena cava (IVC), and traverses the diaphragm via the caval orifice. The vagus nerve gives off the recurrent laryngeal nerve. 28. A patient with a history of stable angina undergoes a coronary angiography. This shows triple vessel disease. What is the usual treatment for such a patient? Select one answer only.

Angioplasty« YOUR ANSWER

Angioplasty and stent insertion

Coronary artery bypass grafting« CORRECT ANSWER

Medical treatment

Surveillance. According to the Coronary Artery Surgery Study the patient groups that derive particular benefit from coronary artery bypass grafting (CABG) are those with triple vessel disease, and those with >50% left main stem stenosis. Those with single or double vessel disease are usually more amenable to percutaneous intervention. Post-myocardial infarction, unstable angina is a primary indication for urgent CABG. Valvular disease with concomitant coronary artery disease is usually treated operatively. 29. During a difficult oesophagectomy in a 55-year-old man there is concern the thoracic duct may have been injured in the mediastinum. Which of the following descriptions best describes the route of the thoracic duct within the mediastinum in relation to the oesophagus? Select one answer only.

It passes anterior and to the left of the oesophagus at T5 level« YOUR ANSWER

It passes anterior and to the right of the thoracic duct at T5 level

It passes posterior and to the left of the oesophagus at T5 level« CORRECT ANSWER

It passes posterior and to the left of the oesophagus at T10 level

It passes posterior and to the right of the thoracic duct at T5 level. The thoracic duct begins below the diaphragm as the cysternachyli and then ascends through the aortic opening in the diaphragm, to the right of the descending aorta. It passes behind the oesophagus and then to the left of the oesophagus at the level of T5. It then runs upwards on the left side of the oesophagus into the neck. Here it crosses the subclavian artery to enter the left brachiocephalic vein. At the root of the neck, the thoracic duct receives the left jugular, subclavian and bronchomediastinal lymph trunks, although they may occasionally drain directly into the adjacent large vessels. The thoracic duct therefore conveys all the lymph from the lower limbs, pelvic cavity, abdominal cavity, left side of the thorax, head and neck and the left arm. 30. In the clinical examination of the chest, accurate knowledge of the surface markings of the

lungs is essential. Which of the following corresponds to the clinical situation? Single best answer

question – choose ONE true option only.

MyPasTest: MRCS A Online - Jan Exam 2015 02. Anatomy; Thorax(120Qs)

-----------------------------------------------------------------------------------------------------------------

------------------------------------------------------------------------------------------------------------------------------------------------------------------------- Dr Mohammed Shamsul Islam Khan, Medical Officer, Clinical Neuro-Surgery, National Institute of Neuro-Sciences and Hospital Sher-E-Bangla

Nagar, Dhaka-1207, Bangladesh. Mobile: +880 1713 455 662, E-mail: [email protected]

Page |

84

The apex of the lung corresponds precisely to the upper border of the medial third of the clavicle« YOUR ANSWER

The oblique fissure of the lung corresponds to the medial border of the scapula when the arm is fully abducted« CORRECT ANSWER

The transverse (horizontal) fissure of the right lung corresponds to the right fifth intercostal space

The lower border of the lung on each side corresponds to the tenth rib in the mid-axillary line

The lower border of the lung reaches the twelfth rib posteriorly. The apex of the lung extends about 4 cm above the medial one-third of the clavicle. The oblique fissure does indeed correspond closely to the medial border of the scapula when the arm is fully abducted. The transverse fissure of the right lung corresponds to the level of the fourth rib. The lower border of the lung on each side corresponds to the eighth rib in the mid-axillary line and the tenth rib posteriorly.

31. A 19-year-old male is brought into A & E following a stabbing. On examination a wound is seen in the left supraclavicular fossa, but he is otherwise haemodynamically stable. A chest X-ray shows the left hemidiaphragm is elevated and there is concern that the phrenic nerve may be affected. Which of the following descriptions best describe the route by which the phrenic nerve enters the chest? Select one answer only.

Anterior to subclavian artery and posterior to the subclavian vein. Medial to the lower border of scalenus anterior« YOUR ANSWER (Correct)

Anterior to the subclavian artery and posterior to the subclavian vein. Lateral to the lower border of scalenus anterior

Anterior to the subclavian vein and artery. Medial to the lower border of scalenus anterior

Posterior to subclavian artery and vein. Medial to the lower border of scalenus anterior

Posterior to subclavian vein and artery. Lateral to the lower border of scalenus anterior. The autonomic fibres in the phrenic nerve are sympathetic and pass from the superior (C1–C4) and middle (C5/6) sympathetic cervical ganglia as grey rami into the C3–C5 roots of the phrenic nerve, and innervate blood vessels in the diaphragm. The nerve lies on the fibrous pericardium and is sensory to the mediastinal and diaphragmatic pleura, and also to the diaphragmatic peritoneum. The phrenic nerve enters the chest by descending from the medial lower border of the scalenus anterior muscle between the subclavian vein anteriorly and artery posteriorly. 32. You are called to the emergency room to see a fifty-six year old female in acute respiratory distress. The A&E registrar informs you that the patient presented with severe sepsis and so a central line has been introduced to optimise emergency management. However, they remain concerned by the patient‟s tachypnoea, cyanosis and falling oxygen saturations. On examination you notice that there is hyperresonance and absent breath sounds in the right hemithorax. What is the most likely diagnosis?

Flail chest« YOUR ANSWER

Massive haemothorax

MyPasTest: MRCS A Online - Jan Exam 2015 02. Anatomy; Thorax(120Qs)

-----------------------------------------------------------------------------------------------------------------

------------------------------------------------------------------------------------------------------------------------------------------------------------------------- Dr Mohammed Shamsul Islam Khan, Medical Officer, Clinical Neuro-Surgery, National Institute of Neuro-Sciences and Hospital Sher-E-Bangla

Nagar, Dhaka-1207, Bangladesh. Mobile: +880 1713 455 662, E-mail: [email protected]

Page |

85

Open pneumothorax

Pleural effusion

Tension pneumothorax« CORRECT ANSWER. The combination of hyperresonance and reduced air entry should always be ascribed to a tension pneumothorax until proven otherwise. Such pneumothoraces are known to be a complication of internal jugular venous catheterisation. Immediate decompression is indicated and should be performed prior to any further interventions.

33. A 67-year-old man undergoes carotid endarterectomy having previously suffered a stroke. During the procedure he sustains an iatrogenic injury to his left vagus nerve. Which of the following statements about the vagus nerve is the most accurate? Select one answer only.

It enters the abdomen at the level of T10« YOUR ANSWER (Correct)

It exits the skull through the foramen ovale

It lies anterior to the hilum of the lung

It originates from the pons

It travels behind of the oesophagus. The vagus nerve arises from the medulla and exits the skull through the jugular foramen. It travels in the carotid sheath between the internal jugular vein and the common carotid artery in the neck. In the chest it lies posterior to the hilum of the lung, whilst the phrenic nerve travels anterior to the hilum. The left vagus travels on the anterior aspect of the oesophagus, whilst the right one travels on the posterior side. Both left and right nerves enter the abdominal cavity at the level of T10 (oesophageal opening). 34. A patient with a malignant mesothelioma is to undergo pleuropneumonectomy, which involves removal of the entire pleura and lung on the affected side. Which of the following layers provides a natural cleavage plane for surgical separation of the costal pleura from the thoracic wall? Single best answer question – choose ONE true option only.

Deep fascia« YOUR ANSWER

Endothoracic fascia« CORRECT ANSWER

Parietal pleura

Visceral pleura

Transversus thoracis muscle fascia. The endothoracic fascia is the connective tissue between the inner aspect of the chest wall and the costal parietal pleura. By clearing the endothoracic fascia, it is easy to separate the costal pleura from the thoracic wall. Deep fascia is a fascial layer that invests a muscle or muscle group - it is not present around the lungs.

MyPasTest: MRCS A Online - Jan Exam 2015 02. Anatomy; Thorax(120Qs)

-----------------------------------------------------------------------------------------------------------------

------------------------------------------------------------------------------------------------------------------------------------------------------------------------- Dr Mohammed Shamsul Islam Khan, Medical Officer, Clinical Neuro-Surgery, National Institute of Neuro-Sciences and Hospital Sher-E-Bangla

Nagar, Dhaka-1207, Bangladesh. Mobile: +880 1713 455 662, E-mail: [email protected]

Page |

86

The parietal pleura comprise the cupola or cervical pleura, costal pleura, diaphragmatic pleura and mediastinal pleura. It lines the inner surfaces of the walls of the pleural cavity. The visceral pleura are the serous membrane that covers the lungs. Finally, the transversus thoracis muscle fascia is only associated with the transversus thoracis - it would not provide a natural cleavage plane for separating the costal pleura from the thoracic wall. 35. A patient presents with a right bundle branch block due to blockage in the atrioventricular nodal artery. Part of the right bundle branch of the arterioventricular bundle is carried by which structure? Single best answer question – choose ONE true option only.

Pectinate muscles« YOUR ANSWER

Anterior papillary muscle of the left ventricle

Moderator band (septomarginal trabecula)« CORRECT ANSWER

Crista terminalis

Chordae tendineae. A muscular band, well marked in sheep and some other animals, frequently extends from the base of the anterior papillary muscle to the ventricular septum. From its attachments it was thought to prevent overdistension of the ventricle and was named the „moderator band‟. However, more recent research has indicated that it is more properly considered part of the electrical conduction system of the heart and in that capacity it is now called the septomarginal trabecula. 36. Following complex surgery for a bypass graft from the right subclavian artery to the brachial artery, a patient is noted to have a hoarse voice post-operatively. Which of the following structures is most likely to have been damaged? Select one answer only.

Brachial plexus« YOUR ANSWER

Left recurrent laryngeal nerve

Phrenic nerve

Right recurrent laryngeal nerve« CORRECT ANSWER

Vagus nerve. The subclavian artery is closely related to the brachial plexus, the phrenic and vagus nerves (and recurrent laryngeal nerve on the right side only). The stellate ganglion is deeper within the neck and the thoracic duct lies on the left side. The approach to the subclavian artery may be supraclavicular and involve removal or division of the clavicle, dissection of the scalenus anterior, pectoralis major and pectoralis minor muscles.

37. You are concerned about myocardial ischaemia in a post-operative patient who is complaining of chest pain. The ECG shows tachycardia and isolated ST-segment depression in leads V1 and V2. Which coronary artery is most likely to be responsible? Single best answer question – choose ONE true option only.

Circumflex coronary artery« YOUR ANSWER

Left anterior descending coronary artery

MyPasTest: MRCS A Online - Jan Exam 2015 02. Anatomy; Thorax(120Qs)

-----------------------------------------------------------------------------------------------------------------

------------------------------------------------------------------------------------------------------------------------------------------------------------------------- Dr Mohammed Shamsul Islam Khan, Medical Officer, Clinical Neuro-Surgery, National Institute of Neuro-Sciences and Hospital Sher-E-Bangla

Nagar, Dhaka-1207, Bangladesh. Mobile: +880 1713 455 662, E-mail: [email protected]

Page |

87

Posterior descending coronary artery« CORRECT ANSWER

Left main stem coronary artery

Right coronary artery. The coronary artery anatomy is variable, as are the relative territories of myocardium supplied by each of the arteries. ST-segment depression across all leads implies ischaemia (angina), whereas isolated ST-segment depression in leads V1 and V2 may indicate a posterior myocardial infarction. The most likely involved artery in this case is therefore the posterior descending artery. Either the right coronary or circumflex arteries can supply the posterior descending artery, but it is not possible to identify which from the ECG alone. Left main stem and left anterior descending artery lesions would cause changes over a much larger area. 38. A 52-year-old morbidly obese diabetic is referred with episodes of chest tightness on minimal exertion. His exercise ECG shows ST depression, and an angiogram demonstrates left main stem stenosis. Which of the following best describes the classical anatomy of the left coronary artery? Select one answer only.

Divides into the circumflex and AV nodal branch« YOUR ANSWER

Divides into the circumflex and left anterior descending artery« CORRECT ANSWER

Divides into the circumflex and sinoatrial nodal branch

Divides into the posterior descending artery and anterior descending artery

Divides into the posterior descending artery and marginal artery. The left coronary artery arises from the left posterior aortic sinus behind the pulmonary trunk. After a short course it divides into two main arteries, the circumflex and the left anterior descending, otherwise known as the anterior interventricular artery. Around 60% of hearts have the right coronary artery supplying the SA node and in 40% of hearts the SA nodal artery arises from the left coronary artery. The right coronary artery supplies the atrioventricular (AV) node. The left coronary artery supplies the vast majority of the left ventricle and left atrium. Part of the right ventricle is supplied by the left coronary artery. 39. A 67-year-old male who is a smoker presents with central chest pain radiating to his neck. His ECG shows ST elevation in chest leads V5 and V6. A coronary angiography shows occlusion of the circumflex artery and, following successful angiography, this can be seen to give rise to the posterior interventricular branch. In approximately what proportion of people is this pattern seen? Select one answer only.

1%« YOUR ANSWER

15%« CORRECT ANSWER

30%

60%

90%.

MyPasTest: MRCS A Online - Jan Exam 2015 02. Anatomy; Thorax(120Qs)

-----------------------------------------------------------------------------------------------------------------

------------------------------------------------------------------------------------------------------------------------------------------------------------------------- Dr Mohammed Shamsul Islam Khan, Medical Officer, Clinical Neuro-Surgery, National Institute of Neuro-Sciences and Hospital Sher-E-Bangla

Nagar, Dhaka-1207, Bangladesh. Mobile: +880 1713 455 662, E-mail: [email protected]

Page |

88

The interventricular septum receives its blood supply from the descending branches from the left coronary (LCA) and right coronary arteries (RCA). RCA travels in the coronary groove to reach the posterior surface of the heart where it anastomoses with the circumflex branch of LCA. Posterior interventricular artery, that is a branch of RCA, anastomoses with the anterior interventricular artery in the interventricular groove. In 15% of hearts the LCA is said to be dominant in that the posterior interventricular branch comes off the circumflex branch. 40. A 27-year-old male is brought into A and E following a stabbing to his left chest. Absent air entry is noted on examination of his left chest and it is dull to percussion. The diagnosis of a haemothorax is made and a chest drain placed. This drains 1600ml of blood immediately and the patient becomes more hypotensive. What should be the next step in managing the patient? Select one answer only.

Blood transfusion and correct any coagulopathy« YOUR ANSWER

Clamp the drain

Emergency thoracotomy« CORRECT ANSWER

Inotropes to improve the blood pressure

Midline sternotomy. Clamping of chest drains is largely to be avoided and achieves nothing clinically. Massive bleed is an immediate indication for urgent thoracotomy for surgical haemostasis (http://www.trauma.org/archive/thoracic/EDTindications.html ). Any haemodynamic instability should lead to consideration of surgical management. Persistent haemorrhage of at least 200 ml/h for four or more hours is an indication for thoracotomy. Video-assisted thoracoscopy may be useful to establish a source of bleeding. Surgical intervention is best undertaken in the operating theatre – a controlled environment, aseptic, adequate light etc. Outcomes from emergency room thoracotomy are poor. Retained blood may cause an empyema. 41. A 61-year-old heavy smoker presents with recurrent chest infections treated with antibiotics from the GP. A mass is seen on his chest X-ray and a bronchoscopy confirms this is malignant in nature. Which of the following is a contraindication to resection? Select one answer only.

COPD« YOUR ANSWER

FEV/FVC of 70%

Ipsilateralhilar lymphadenopathy

Malignant pleural effusion « CORRECT ANSWER

Ongoing smoking status. A lung tumour must be sufficiently localised to be suitable for resection. Involvement of ipsilateralhilar lymph nodes is not usually a contraindication to resection but the presence of malignant cells in a pleural effusion is. The 5-year survival rate following complete resection of non-small-cell lung cancer is in the region of 30–40%.

MyPasTest: MRCS A Online - Jan Exam 2015 02. Anatomy; Thorax(120Qs)

-----------------------------------------------------------------------------------------------------------------

------------------------------------------------------------------------------------------------------------------------------------------------------------------------- Dr Mohammed Shamsul Islam Khan, Medical Officer, Clinical Neuro-Surgery, National Institute of Neuro-Sciences and Hospital Sher-E-Bangla

Nagar, Dhaka-1207, Bangladesh. Mobile: +880 1713 455 662, E-mail: [email protected]

Page |

89

Small-cell lung cancers (also known as oat-cell carcinoma) are highly malignant tumours that are usually disseminated at presentation. For the majority of patients chemotherapy is the treatment of choice, small-cell lung cancers are rarely suitable for surgical management. 42. A 35-year-old woman is noted to have a left phrenic nerve injury after excision of a left cervical rib. Which of these descriptions best describes the relation of the phrenic nerve in this area? Select one answer only.

Anterior to scalenus anterior« YOUR ANSWER (Correct)

Between scalenus anterior and scalenus medius

Medial to scalenus anterior

Posterior to scalenus medius

Posterior to scalenus posterior. The phrenic nerve arises from the spinal cord segments C3–5 and lies in front of the scalenus anterior muscle, passing between the subclavian vein anteriorly and the subclavian artery posteriorly. It crosses over the lateral surfaces of the mediastinal structures passing in front of the lung root to reach the diaphragm. The phrenic nerve supplies motor fibres to the diaphragm and carries sensory fibres from the diaphragmatic peritoneum, mediastinal pleura and the parietal pericardium. The bronchi and lungs themselves are supplied by branches of the autonomic nerves, principally via the pulmonary plexuses. 43. A 56-year-old diabetic male is undergoing a CABG for left main stem disease following an MI. Which of the following terms best describes the anaesthetic approach to cardiac surgery of this nature? Select one answer only.

Controlled hypertension, controlled hyperthermia« YOUR ANSWER

Controlled hypertension, controlled hypothermia

Controlled hypotension, controlled hypothermia« CORRECT ANSWER

Controlled hypotension, controlled normothermia

Controlled normotension, controlled hypothermia. Cardiac surgery is performed with controlled hypotension, together with hypothermia. The hypothermia is used to decrease cellular metabolism and reduce energy requirements of the tissues. Cardiac tamponade is a well known complication of cardiac surgery, it usually presents in the early post-operative period with deteriorating cardiac function and cardiac arrest. CABG can sometimes be performed without cardiopulmonary bypass („off pump‟). Arrhythmias (usually atrial fibrillation) occur in 20–40% of patients following surgery. Routine CABG is associated with a mortality rate of around 2%.

MyPasTest: MRCS A Online - Jan Exam 2015 02. Anatomy; Thorax(120Qs)

-----------------------------------------------------------------------------------------------------------------

------------------------------------------------------------------------------------------------------------------------------------------------------------------------- Dr Mohammed Shamsul Islam Khan, Medical Officer, Clinical Neuro-Surgery, National Institute of Neuro-Sciences and Hospital Sher-E-Bangla

Nagar, Dhaka-1207, Bangladesh. Mobile: +880 1713 455 662, E-mail: [email protected]

Page |

90

44. A 23-year-old motorcyclist presents with a blunt chest injury and is noted to have hyperresonance and absent breath sounds over the right hemithorax with evidence of tracheal deviation to the left. What should be the first clinical intervention?

Endotracheal intubation « YOUR ANSWER

Intercostal drainage

Needle decompression« CORRECT ANSWER

Positive end-expiratory pressure ventilation

Thoracotomy. This patient has a right-sided tension pneumothorax. If such a diagnosis is suspected, immediate needle decompression should be performed by inserting a wide bore cannula into the second intercostal space in the mid-clavicular line of the affected hemithorax. This „buys time‟ for the more definitive intervention of intercostal drain insertion. Positive end-expiratory pressure ventilation is known to be a risk factor for the development of a tension pneumothorax. Thoracotomy is rarely indicated in cases of tension pneumothorax. 45. A 19-year-old male gang member is brought into A&E after being stabbed multiple times in the right chest. He is drowsy and has a BP on arrival of 70/40 with cool peripheries. The right side of his chest is dull to percussion with absent breath sounds. A chest drain is placed in his right chest and drains 1700ml within 15 minutes. What is the diagnosis? Select one answer only.

Haemothorax« YOUR ANSWER

Haemopneumothorax

Massive haemothorax« CORRECT ANSWER

Pneumothorax

Tension pneumothorax. A haemothorax may result from a blunt (deceleration injury) or penetrating injury (disruption of the systemic or hilar vessels) to the thoracic cavity. Massive haemothorax results from the rapid accumulation of more than 1500 ml of blood or one-third or more of the patient‟s blood volume in the chest cavity. Distension of neck veins is rare; they are usually flat secondary to severe hypovolemia. Rarely will the mechanical effects of massive intrathoracic blood shift the mediastinum enough to cause distended neck veins or a tracheal shift. The neck veins, however, may be distended if there is an associated tension pneumothorax. The important signs and symptoms of a massive haemothorax include hypoxia, chest pain, decreased chest expansion, absence of breath sounds on the affected side and percussion dullness over the affected hemithorax. Chest tube placement to decompress the chest cavity, along with simultaneous restoration of blood volume, is the first step in the management of massive traumatic haemothorax. Blood from the chest tube should be collected in a device suitable for auto-transfusion. If 1500 ml is immediately evacuated, it is highly likely that the patient will require an early thoracotomy. In

MyPasTest: MRCS A Online - Jan Exam 2015 02. Anatomy; Thorax(120Qs)

-----------------------------------------------------------------------------------------------------------------

------------------------------------------------------------------------------------------------------------------------------------------------------------------------- Dr Mohammed Shamsul Islam Khan, Medical Officer, Clinical Neuro-Surgery, National Institute of Neuro-Sciences and Hospital Sher-E-Bangla

Nagar, Dhaka-1207, Bangladesh. Mobile: +880 1713 455 662, E-mail: [email protected]

Page |

91

addition, patients who have an initial volume output of less than 1500 ml but continue to bleed (200 ml/h for 2–4 h) also require a thoracotomy. The decision should be made in such patients based on their physiological status rather than the rate of continuing blood loss. The majority of the patients can be managed conservatively with appropriate fluid resuscitation and chest decompression. Emergency thoracotomy is required in only about 10% of patients with massive haemothorax. 46. A 32-year-old male is rushed into A & E following an explosion at a factory. He has severe facial burns and requires a needle cricothyroidotomy following a failed intubation. Following stabilisation, it is decided to perform a tracheostomy. Which of the following landmarks represents the origin of the trachea? Select one answer only.

Cricoid cartilage« YOUR ANSWER (Correct)

Hyoid bone

Jugular notch

Laryngeal prominence

Sternal angle. The trachea commences at the lower border of the cricoid cartilage, is palpable in the jugular notch, bifurcates in the transthoracic plane and is innervated by the recurrent laryngeal nerves. The left main bronchus bifurcates inside the left lung and is not as vertical as the right main bronchus. 47. A 41-year-old male is brought into A & E very short of breath following a road traffic accident. He is unable to complete sentences and has a respiratory rate of 34. On examination there is bruising over his right chest and chest expansion on the right is reduced with tracheal deviation away from this side. Percussion is resonant and breath sounds are normal on the left, but on the right percussion is hyper-resonant and breath sounds are absent. What is the most likely diagnosis? Select one answer only.

Flail chest« YOUR ANSWER

Haemothorax

Pneumothorax

Tension pneumothorax« CORRECT ANSWER

Chylothorax. Tension pneumothorax is one of the immediate life-threatening injuries and must be diagnosed and

managed very early. It is recognised by respiratory distress, tracheal deviation away from the

affected side, unilateral absent breath sounds, distended neck veins and not always accompanied

by rib fractures.

48. A 73-year-old man is found to have a thoracic aortic aneurysm affecting the ascending aorta and aortic arch. The aneurysm is expanding in size, leading to aortic regurgitation. Which of the following best describes the anatomy of the normal thoracic aortic arch? Select one answer only.

MyPasTest: MRCS A Online - Jan Exam 2015 02. Anatomy; Thorax(120Qs)

-----------------------------------------------------------------------------------------------------------------

------------------------------------------------------------------------------------------------------------------------------------------------------------------------- Dr Mohammed Shamsul Islam Khan, Medical Officer, Clinical Neuro-Surgery, National Institute of Neuro-Sciences and Hospital Sher-E-Bangla

Nagar, Dhaka-1207, Bangladesh. Mobile: +880 1713 455 662, E-mail: [email protected]

Page |

92

It arches below the manubriosternal joint« YOUR ANSWER

It arches directly over the right pulmonary artery

It gives rise to the coronary arteries

It is covered by pleura« CORRECT ANSWER

It lies anterior to the brachiocephalic veins. The apex of the arch, which gives attachment to the pretracheal fascia, lies posteroinferior to the left brachiocephalic vein. The lower border of the arch lies in the transthoracic plane and on the left is directly related to the left pulmonary artery (the superior vena cava lies over the right pulmonary artery). The arch is symmetrically covered by the pleura from both sides, which meet in the midline behind the manubriosternal joint. The coronary arteries are the first branches of the ascending aorta and not branches of the aortic arch. This gives rise to the brachiocephalic artery, the left common carotid and the left subclavian artery. 49. During angiography a 56-year-old diabetic patient is noted to have stenosis of the posterior interventricular artery. Which of the following most often gives rise to this vessel? Select one answer only.

Anterior interventricular artery« YOUR ANSWER

Circumflex artery

Left coronary artery

Marginal branch of the right coronary artery

Right coronary artery« CORRECT ANSWER. The right coronary artery runs in the atrioventricular groove between the right atrium and right

ventricle. It has no branches which directly supply the ventricular septum. The posterior

descending artery most commonly arises from the right coronary artery, and can be found

overlying the interventricular septum posteriorly.

It may arise as a branch of the circumflex artery, but not as a branch of the left anterior descending

artery. The main circumflex artery does lie in the atrioventricular groove, and its obtuse marginal

branches supply the lateral wall of the left ventricle. The acute marginal branch arises from the right

coronary artery.

50. A 72-year-old smoker undergoes a resection of the right middle lung lobe for squamous cell lung cancer. The lesion is adherent to the mediastinum but is removed successfully. A post-operative chest x-ray reveals a raised right hemi-diaphragm. There is no evidence of a pneumothorax. Which of the following structures is most likely to have been damaged? Select one answer only.

The liver capsule and parenchyma« YOUR ANSWER

The phrenic nerve« CORRECT ANSWER

MyPasTest: MRCS A Online - Jan Exam 2015 02. Anatomy; Thorax(120Qs)

-----------------------------------------------------------------------------------------------------------------

------------------------------------------------------------------------------------------------------------------------------------------------------------------------- Dr Mohammed Shamsul Islam Khan, Medical Officer, Clinical Neuro-Surgery, National Institute of Neuro-Sciences and Hospital Sher-E-Bangla

Nagar, Dhaka-1207, Bangladesh. Mobile: +880 1713 455 662, E-mail: [email protected]

Page |

93

The right hemi-diaphragm

The thoracic duct

The vagus nerve. The phrenic nerve (C3-5) provides sensory and motor fibres to the diaphragm, as well as fibres to the pleura and mediastinal pericardium. An injury of the phrenic nerve causes paralysis of the corresponding hemidiaphragm. This leads to paradoxical movement of the diaphragm, rising on inspiration. The phrenic nerve may be injured in any surgical procedure in the neck (where it lies on the anterio scalene muscle) or near the mediastinum. Spinal cord injuries above or at the level of C3-C5 may lead to respiratory compromise. Remember 'C3, 4 and 5 keep the diaphragm alive'. A raised hemidiaphragm may also be caused by a subphrenic abscess or more rarely by a subcapsular haematoma of the liver. In the context of recent thoracic surgery, the most likely cause is iatrogenic phrenic nerve damage during the tumour dissection. 51. A 63-year-old long term smoker presents with haemoptysis and weight loss. A chest X-ray shows a tumour and bronchoscopy reveals it to be at the carina. At what level is the carina? Select one answer only.

Level with xiphoid process« YOUR ANSWER

Jugular notch

Sternal angle of Louis« CORRECT ANSWER

Suprasternal notch

Transpyloric plane. The brachiocephalic artery bifurcates behind the right sternoclavicular joint. The ascending aorta becomes the aortic arch at the manubriosternal joint (sternal angle of Louis). A plane passing posteriorly will reach the lower border of the T4 vertebrae dividing the mediastinum into the superior mediastinum and inferior mediastinum. The trachea bifurcates at the carina into the two main stem bronchi at this level. At this level, the azygous, the left recurrent laryngeal nerve looping around the ligamentumarteriosum, and the bifurcation of the pulmonary trunk, can also be seen. 52. A 71-year-old heavy smoker is listed for a left posterolateral thoracotomy for resection of a tumour. How should he be positioned on the table? Select one answer only.

Left lateral position« YOUR ANSWER

Prone

Reverse trendelenburg

Right lateral position« CORRECT ANSWER

Supine. Posterolateral thoracotomy is performed most commonly with the patient in a full lateral position.

The skin incision extends from a point two fingerbreadths below the inferior angle of the scapula,

MyPasTest: MRCS A Online - Jan Exam 2015 02. Anatomy; Thorax(120Qs)

-----------------------------------------------------------------------------------------------------------------

------------------------------------------------------------------------------------------------------------------------------------------------------------------------- Dr Mohammed Shamsul Islam Khan, Medical Officer, Clinical Neuro-Surgery, National Institute of Neuro-Sciences and Hospital Sher-E-Bangla

Nagar, Dhaka-1207, Bangladesh. Mobile: +880 1713 455 662, E-mail: [email protected]

Page |

94

posteriorly to a point halfway between the spine of the scapula and the vertebral column, and

anteriorly following the line of the rib for a variable distance.

The latissimus dorsi is routinely divided but the serratus anterior is usually spared. The chest is

entered through the intercostal spaces, which are counted as starting below the first rib. Thus the

fifth space lies below the fifth rib. The intercostal muscles are divided from the superior border of

the rib below the intercostal space, to be entered to avoid injury to the neurovascular bundle lying

just inferior to each rib.

53. The sinus venosus is the large quadrangular cavity located between the two vena cava vessels in the embryonic human heart. In the adult heart the sinus venosus gives rise to the: Single best answer question – choose ONE true option only.

Aortic vestibule of the left ventricle« YOUR ANSWER

Coronary sinus« CORRECT ANSWER

Fossa ovalis

Trabeculated part of the right ventricle

Trabeculated portion of the right atrium. The derivatives of the embryonic sinus venosus include: the smooth part of the right atrium (sinus venarum), the „valve‟ of the superior vena cava and the sinoatrial node from the right horn; the coronary sinus and the valve of coronary sinus from the left horn; the border of smooth part of right atrium (crista terminalis) from the right half of the valve of the sinus venosus and part of the atrial septum from the left half of the valve of the sinus venosus. 54. You are asked by your consultant to perform a median sternotomy under supervision. During the manoeuver you are asked to describe which vascular structures are positioned on the same anatomical plane as the manubriosternal joint. What is the correct answer?

Aortic arch and junction of the azygos vein and hemiazygos vein« YOUR ANSWER

Aortic arch and junction of the azygos vein and superior vena cava« CORRECT ANSWER

Descending thoracic aorta and junction of the azygos and superior vena cava

Hemiazygos vein

Origin of the brachiocephalic artery. The manubriosternal joint marks the thoracic plane, which is at the level of the T4/5 intervertebral disc. Other structures found on this plane include the aortic arch, the junction of the azygos vein with the SVC, the 2nd costal cartilages, the tracheal bifurcation, the ligamentum arteriosum and the decussation of the thoracic duct. The manubriosternal joint is also known as the Angle of Louis. 55. A 78-year-old male being worked up for an open abdominal aortic aneurysm repair undergoes a routine chest X-ray which shows a widened mediastinum. Further imaging confirms a thoracic aneurysm affecting the aortic arch. Which of the following branches normally arises from the aortic arch? Select one answer only.

MyPasTest: MRCS A Online - Jan Exam 2015 02. Anatomy; Thorax(120Qs)

-----------------------------------------------------------------------------------------------------------------

------------------------------------------------------------------------------------------------------------------------------------------------------------------------- Dr Mohammed Shamsul Islam Khan, Medical Officer, Clinical Neuro-Surgery, National Institute of Neuro-Sciences and Hospital Sher-E-Bangla

Nagar, Dhaka-1207, Bangladesh. Mobile: +880 1713 455 662, E-mail: [email protected]

Page |

95

Left internal mammary artery« YOUR ANSWER

Left subclavian artery« CORRECT ANSWER

Right common carotid artery

Right subclavian artery

Right vertebral artery. The arch of the aorta commences from the manubriosternal joint and passes backwards over the left bronchus to reach the body of T4 vertebra just to the left of the midline. The branches arising directly from the aortic arch are the brachiocephalic trunk (which will divide into the right subclavian artery and right common carotid artery), the left common carotid artery and left subclavian artery. The arch is crossed on its left side by the phrenic and vagus nerves as they pass downwards in front of and behind the lung root respectively. The left vertebral artery and the right internal mammary artery come off the left and right subclavian arteries respectively. 56. A 21-year-old man is brought into A&E unresponsive after being stabbed in the chest. On examination the knife has been left in place and is entering through the 4th intercostal space on the left of the sternum. He has a heart rate of 140 and his BP is 70/30. He is noted to have a central trachea, distended neck veins. He has normal breath sounds bilaterally, but muffled heart sounds. What is the most likely diagnosis? Select one answer only.

Aortic dissection« YOUR ANSWER

Cardiac tamponade« CORRECT ANSWER

Haemothorax

Pneumothorax

Tension pneumothorax. ECG is useful to detect dysrhythmias and pulseless electrical activity (PEA). There is no indication for routine exploratory thoracotomy within 1 hour. An urgent exploratory thoracotomy should be performed if 1500 ml of blood is drained immediately from the chest drain or 200 ml/h for 2–4 hours. A normal appearance of the cardiac shadow on CXR is insufficient to exclude a tamponade. Echocardiography and computed tomography (CT) are more sensitive methods. Neck veins should be assessed for distension. Beck‟s triad for cardiac tamponade consists of muffled heart sounds, hypotension and distended neck veins. Tension pneumothorax is a clinical diagnosis requiring immediate management. Aortic dissection classically causes a tearing chest pain radiating through to the back. On examination, the patient can have a variety of symptoms depending on which part of the aorta (and therefore who branches) are affected. If the common carotid artery is affected they may have a neurological signs, there may be signs in the upper limb if the subclavian arteries are affected.

MyPasTest: MRCS A Online - Jan Exam 2015 02. Anatomy; Thorax(120Qs)

-----------------------------------------------------------------------------------------------------------------

------------------------------------------------------------------------------------------------------------------------------------------------------------------------- Dr Mohammed Shamsul Islam Khan, Medical Officer, Clinical Neuro-Surgery, National Institute of Neuro-Sciences and Hospital Sher-E-Bangla

Nagar, Dhaka-1207, Bangladesh. Mobile: +880 1713 455 662, E-mail: [email protected]

Page |

96

If the dissection involves the abdominal aorta, the patient may have abdominal signs or oliguria/ anuria if the renal arteries are affected. Classically, there may be a difference in blood pressure between the arms due to subclavian artery involvement on one side. A haemothorax would cause chest signs on examination- reduced or absent air/entry and dullness to percussion, this helps to differentiate it from a pneumothorax in which air entry may also be reduced or absent, but percussion is hyper-resonant. In tension pneumothorax a valve like mechanism allows the accumulation of air in the pleural space which causes medistinal displacement- thus affecting venous return to the heart and therefore cardiac output. It will lead to severe shortness of breath, tracheal deviation, dilated neck veins and ultimately cardiac arrest. The diagnosis is clinical, and management should not be delayed for a chest X ray. 57. An 82-year old man develops hilar lymphadenopathy secondary to a haematological malignancy. This affects predominantly the superior aspect of the right main bronchus. Which of the following structures is most likely to be compressed? Select one answer only.

The azygos vein« YOUR ANSWER (Correct)

The hemiazygos nerve

The right phrenic nerve

The right recurrent laryngeal nerve

The right vagus nerve. The vagus nerve lies just posterior to the right main bronchus and the azygos vein is at first posterior and then arches over the superior aspect of the right main bronchus to join the SVC. The phrenic nerve is anterior to the bronchus. The right recurrent laryngeal nerve hooks around the right subclavian artery superior to the right main bronchus. 58. A 78-year-male with severe aortic stenosis undergoes coronary artery bypass grafting for coronary artery disease whilst having an aortic valve replacement. Which is the most commonly used arterial conduit? Select one answer only.

Inferior epigastric artery« YOUR ANSWER

Left internal mammary artery« CORRECT ANSWER

Long saphenous vein

Right internal mammary artery

Radial artery. The most commonly used arterial conduit is the left internal mammary artery. Currently the right

internal mammary artery and radial artery are also in use by some surgeons. The gastroepiploic

and inferior epigastric arteries have been used, but are uncommon bypass conduits.

MyPasTest: MRCS A Online - Jan Exam 2015 02. Anatomy; Thorax(120Qs)

-----------------------------------------------------------------------------------------------------------------

------------------------------------------------------------------------------------------------------------------------------------------------------------------------- Dr Mohammed Shamsul Islam Khan, Medical Officer, Clinical Neuro-Surgery, National Institute of Neuro-Sciences and Hospital Sher-E-Bangla

Nagar, Dhaka-1207, Bangladesh. Mobile: +880 1713 455 662, E-mail: [email protected]

Page |

97

59. A victim of anterior chest stabbing received a stab in a structure that is in close proximity to where the first rib articulates with the sternum. The structure most likely to be injured is the: Single best answer question – choose ONE true option only.

Nipple« YOUR ANSWER

Root of the lung

Sternal angle

Sternoclavicular joint« CORRECT ANSWER

Xiphoid process. The first rib articulates with the sternum directly below the sternoclavicular joint. The nipple (in men) may be considered to be in the fourth intercostal space, between the fourth and fifth ribs, but this is an inconsistent surface landmark due to anatomical variation (eg the presence of breasts in women). The sternal angle is connected to the costal cartilage of rib 2. Finally, the xiphoid process is located just below the point where the costal cartilage of rib 7 articulates with the sternum. The root of the lung consists of the main bronchus, pulmonary and bronchial vessels, lymphatic vessels and nerves entering and leaving the lung. The roots of the lungs are anterior to vertebrae T5 (right) and T6 (left). 60. A 48-year-old female smoker presents with acute severe chest pain and ST elevation on the ECG. Troponin levels are raised and suggestive of myocardial infarction. She undergoes Primary Coronary Angioplasty and is noted to have an occlusion within the vessel running through the right coronary sulcus. Which vessel has been occluded?

Conus artery« YOUR ANSWER

Left circumflex artery

Left coronary artery

Posterior descending artery

Right coronary artery« CORRECT ANSWER. The right coronary artery originates from the ascending aorta just distal to the right cusp of the aortic valve. It travels within the right coronary sulcus and gives off the posterior descending artery and sinoatrial node artery in most cases. The conus artery is present in 45% of the population and can provide collateral circulation in cases of left anterior descending artery occlusion. 61. In a post-operative immunosuppressed transplant patient, some small vesicles have formed on the chest wall, in a linear patch, approximately level with his xiphoid process. Which dermatome is most likely to be affected? Single best answer question – choose ONE true option only.

C4« YOUR ANSWER

T4

T7« CORRECT ANSWER

T10

MyPasTest: MRCS A Online - Jan Exam 2015 02. Anatomy; Thorax(120Qs)

-----------------------------------------------------------------------------------------------------------------

------------------------------------------------------------------------------------------------------------------------------------------------------------------------- Dr Mohammed Shamsul Islam Khan, Medical Officer, Clinical Neuro-Surgery, National Institute of Neuro-Sciences and Hospital Sher-E-Bangla

Nagar, Dhaka-1207, Bangladesh. Mobile: +880 1713 455 662, E-mail: [email protected]

Page |

98

L1. C4 corresponds to the lower neck just above the clavicle. T4 corresponds to the level of the

nipples. T7 is the level of the xiphoid process. T10 corresponds to the level of the umbilicus, and L1

corresponds to the lower abdomen / inguinal region.

62. A 22-year-old male is rushed into A & E after being stabbed in the chest following an assault. The knife is still in place and he is taken to theatre for urgent surgical exploration. The knife is found to have penetrated the anterior wall of the right atrium. Which of the following structures are present within the anterior wall of the right atrium? Select one answer only.

Conusarteriosus« YOUR ANSWER

Coronary sinus

Musculipectinati« CORRECT ANSWER

Oval fossa

Sinus venarum. The right atrium forms the right border of the heart between the superior and inferior vena cava.

The internal walls consist of a smooth part posteriorly, the sinus venarum, which receives the

venae cavae and coronary sinus and a rough anterior portion, which has the musculipectinati. The

conusarteriosus is found in the right ventricle.

63. A 32-year-old male who is involved in a road traffic accident whilst driving his motorbike has a pneumothorax clinically which is confirmed on imaging. It is decided he requires a chest drain. Which of the following best describes a safe location for insertion? Select one answer only.

Posterior to the mid-axillary line, 4th intercostal space below 4th rib« YOUR ANSWER

Posterior to the mid-axillary line, 4th intercostal space above 5th rib

Anterior to the mid-axillary line, 4th intercostal space below 4th rib

Anterior to the mid-axillary line, 4th intercostal space above 5th rib« CORRECT ANSWER

2nd intercostal space, midclavicular line. For the anatomy of chest drain insertion, it is important to remember the 'safe triangle.' This is bordered by the anterior border of the latissimus dorsi (mid-axillary line), the lateral border of the pectoralis major muscle, a line superior to the horizontal level of the nipple (5th intercostal space), and an apex below the axilla. For insertion, the area should be draped, and patient consented (as per BTS guidelines). Local anaesthestics can then be infiltrated over the area and a 2cm incision made ABOVE the rib (this is to minimise the risk of damage to the neurovascular bundle), and blunt dissection performed down to the pleura. Forceps or scissors can be used to pierce the pleura and a finger should be swept around inside prior to drain insertion to remove any adherent lung tissue.

MyPasTest: MRCS A Online - Jan Exam 2015 02. Anatomy; Thorax(120Qs)

-----------------------------------------------------------------------------------------------------------------

------------------------------------------------------------------------------------------------------------------------------------------------------------------------- Dr Mohammed Shamsul Islam Khan, Medical Officer, Clinical Neuro-Surgery, National Institute of Neuro-Sciences and Hospital Sher-E-Bangla

Nagar, Dhaka-1207, Bangladesh. Mobile: +880 1713 455 662, E-mail: [email protected]

Page |

99

The drain can then be inserted (without the trocar). When placing a chest drain it should be pointed upward if a pneumothorax is being drained, and downward if fluid e.g. a pleural effusion is being drained. Alternatively some kits nowadays utilise the Seldinger technique for insertion. Following insertion chest drains should be connected with an appropriate underwater seal. It is important to note, they should never be clamped. The 2nd intercostal space, midclavicular line is utilised when performing a needle decompression of a tension pneumothorax to relieve the tensioning. Following this a formal chest drain can be inserted as described above. 64. A 48-year-old woman presents with muscle weakness and problems chewing food. Her symptoms appear to be worsened on repetition and on examination she is noted to have bilateral ptosis, and a diagnosis of myasthenia gravis is made. A CT thorax shows an anterior mediastinal tumour. What is likely to be the nature of this? Select one answer only.

Germ cell tumour« YOUR ANSWER

Lymphoma

Mesothelioma

Small cell lung cancer

Thymoma« CORRECT ANSWER. Thymoma is the commonest anterior mediastinal tumour, although it is still rare in everyday clinical

practice. There is a definite association with myasthenia gravis. The differential diagnosis for an

anterior mediastinal mass is lymphoma and germ cell tumour. It is germ cell tumours that may

cause an elevated serum b-human chorionic gonadotrophin (b-HCG).

Any mass in the anterior mediastinum may compress the superior vena cava leading to the

superior vena cava obstruction syndrome. Needle biopsy is still not useful in differentiating benign

from malignant thymic tumours, but is fairly reliable in differentiating these tumours from

lymphoma and germ cell tumours.

65. A 27-year-old female is brought into A & E as a trauma call after a horse riding accident in which the horse she was riding fell backward landing on the left side of her chest. On arrival she is short of breath and struggling to complete sentences. On examination the chest wall containing ribs 3-5 appears to move in on inspiration, air entry is present bilaterally and her trachea is central. What is the most likely diagnosis? Select one answer only.

Flail chest« YOUR ANSWER (Correct)

Haemothorax

Pneumothorax

Surgical emphysema

MyPasTest: MRCS A Online - Jan Exam 2015 02. Anatomy; Thorax(120Qs)

-----------------------------------------------------------------------------------------------------------------

------------------------------------------------------------------------------------------------------------------------------------------------------------------------- Dr Mohammed Shamsul Islam Khan, Medical Officer, Clinical Neuro-Surgery, National Institute of Neuro-Sciences and Hospital Sher-E-Bangla

Nagar, Dhaka-1207, Bangladesh. Mobile: +880 1713 455 662, E-mail: [email protected]

Page |

100

Tension pneumothorax. Ribs are narrow, flat, curved bones that form the thoracic cage. The rib has an angle at its posterior

end. The costal groove is found on the inferior border of the ribs. The intercostal vessels and nerve

run in this groove. Ribs typically articulate with their corresponding vertebra plus the one above it.

The area just anterior to the angle is the weakest, and therefore the most likely to fracture in

trauma.

66. A 34-year-old, tall man presents with sudden onset shortness of breath. He is noted to have absent breath sounds in the left hemi-thorax. A chest x-ray confirms a spontaneous simple left pneumothorax and chest drain is inserted. When inserting an intercostal drain in the 5th intercostal space. Which of the following structures is normally breached by the chest tube? Select one answer only.

Parietal pleura« YOUR ANSWER (Correct)

Pectoralis major muscle

Pectoralis minor muscle

Pericardium

Visceral pleura. An intercostal drain should be inserted in the fifth intercostal space, anterior to the mid-axillary line and so should not penetrate pectoralis major. The visceral pleura overlies the lung and should not be entered. After piercing the skin, fat and intercostals muscles, the chest drain must pass through the parietal pleura lining the inner thorax. This allows the drain to reach the interpleural space (found between the parietal and visceral pleurae), which allows the pleural cavity to be drained. 67. After a difficult right-sided subclavian central line insertion, which is the least likely structure to be damaged? Single best answer question – choose ONE true option only.

Thoracic duct« YOUR ANSWER (Correct)

Brachial plexus

Subclavian vein

Subclavian artery

Lung apex. The thoracic duct has usually crossed over to the left side by the level of the 5th or 6th vertebral

body, and is therefore the furthest structure from the advancing needle. The brachial plexus,

subclavian artery and lung can all be reached and damaged by a needle during subclavian

cannulation. The subclavian vein, although the target vessel in this case (and therefore intentionally

punctured) can be damaged (torn) during line insertion.

68. A 68-year-old heavy smoker presents to his GP with a drooping left eyelid and dryness affecting the left side of his face. His GP notes he has a left sided ptosis, miosis and anhidrosis, and a

MyPasTest: MRCS A Online - Jan Exam 2015 02. Anatomy; Thorax(120Qs)

-----------------------------------------------------------------------------------------------------------------

------------------------------------------------------------------------------------------------------------------------------------------------------------------------- Dr Mohammed Shamsul Islam Khan, Medical Officer, Clinical Neuro-Surgery, National Institute of Neuro-Sciences and Hospital Sher-E-Bangla

Nagar, Dhaka-1207, Bangladesh. Mobile: +880 1713 455 662, E-mail: [email protected]

Page |

101

diagnosis of Horner syndrome is made. A CT scan shows a Pancoast tumour invading the stellate ganglion. Which of the following descriptions best describes the relations of the stellate ganglion? Select one answer only.

Anterior to the neck of the first rib« YOUR ANSWER (Correct)

Anterior to the scalene tubercle

Lateral to the angle of the first rib

Posterior to the neck of the first rib

Posterior to the scalene tubercle. The scalenus anterior inserts on the scalene tubercule. A groove is found on the first rib anterior to

the scalene tubercule for the subclavian vein. The subclavian artery runs behind the scalenus

anterior muscle. The first rib is related to the lower two roots of the brachial plexus, C8 and T1. The

cervicothoracic ganglion, otherwise known as the „stellate ganglion‟, lies in front of the neck of the

first rib.

69. A 21-year old man is shot in the chest. He is diagnosed with a pericardial tamponade. He undergoes an emergency thoracotomy and creation of a pericardial window by excising a segment of pericardium to relieve the tamponade. Which of the following can occur as result of an iatrogenic injury during the pericardial incision? Select one answer only.

Damage to the parasympathetic supply to the abdomen« YOUR ANSWER

Elevation of the left hemidiaphragm« CORRECT ANSWER

Hiatal hernia

Paralysis of all intercostals muscles on the left hemithorax

Pulsion diverticulum of the oesophagus. The phrenic nerve (C3 – 5) supplies motor innervation to the diaphragm. It runs vertically on the external surface of the cardiac pericardium. An incision over the pericardium in emergency situations can lead to a phrenic nerve damage and paralysis of the ipsilateral hemidiaphragm. 70. A 25-year-old female presents with pain in the right hand which is associated with a tingling sensation and weakness. After further investigation she is diagnosed with thoracic outlet syndrome. The affected neurovascular bundle runs through which muscles?

Digastric« YOUR ANSWER

Platysma

Scalene« CORRECT ANSWER

Sternocleidomastoid

Sternothyroid. Thoracic outlet syndrome is caused by compression of the neurovascular bundle that passes between the anterior and middle scalene muscles. The brachial plexus and the subclavian artery

MyPasTest: MRCS A Online - Jan Exam 2015 02. Anatomy; Thorax(120Qs)

-----------------------------------------------------------------------------------------------------------------

------------------------------------------------------------------------------------------------------------------------------------------------------------------------- Dr Mohammed Shamsul Islam Khan, Medical Officer, Clinical Neuro-Surgery, National Institute of Neuro-Sciences and Hospital Sher-E-Bangla

Nagar, Dhaka-1207, Bangladesh. Mobile: +880 1713 455 662, E-mail: [email protected]

Page |

102

and (rarely) the subclavian vein can be affected, leading to a variety of upper limb, neck and upper back symptoms. Cervical ribs, Pancoast‟s tumour and preganancy can all cause the condition, but it is usually secondary to trauma or repetitive strain injuries. 71. While performing a surgical procedure in the mid-region of the thorax the surgeon accidentally injured an important structure that lies immediately anterior to the thoracic duct. Which one of the following structures was most likely to be injured? Single best answer question – choose ONE true option only.

Aorta« YOUR ANSWER

Azygos vein

Oesophagus« CORRECT ANSWER

Superior vena cava

Trachea. In the mid-thorax, the aorta, thoracic duct and azygos vein are all posterior to the oesophagus. (They are in that order, from left to right.) The superior vena cava and the trachea are not located in the mid-thorax - the superior vena cava terminates as it feeds into the right atrium and the trachea ends as it splits into the two main stem bronchi, which enter the lungs. 72. A 48-year-old female smoker presents with acute severe chest pain and ST elevation on the ECG. Troponin levels are raised and suggestive of myocardial infarction. She undergoes Primary Coronary Angioplasty and is noted to have an occlusion within the short artery that passes between the left atrial appendage and the pulmonary trunk. Which vessel has been occluded?

Conus artery« YOUR ANSWER

Left circumflex artery

Left coronary artery« CORRECT ANSWER

Posterior descending artery

Right coronary artery. The left coronary artery originates from the ascending aorta just distal to the aortic valve within the sinus of Valsalva. It courses between the pulmonary trunk and the left atrial appendage before dividing into the circumflex and left anterior descending (LAD) arteries. In some cases, it fails to develop, and the circumflex and LAD originate from the left aortic sinus. 73. A 32-year-old man was shot in the chest. The bullet punctured a vessel that courses across the mediastinum in an almost horizontal fashion. Which one of the following vessels was injured? Single best answer question – choose ONE true option only.

Left subclavian artery« YOUR ANSWER

Left subclavian vein

Left brachiocephalic vein« CORRECT ANSWER

Left internal jugular vein

MyPasTest: MRCS A Online - Jan Exam 2015 02. Anatomy; Thorax(120Qs)

-----------------------------------------------------------------------------------------------------------------

------------------------------------------------------------------------------------------------------------------------------------------------------------------------- Dr Mohammed Shamsul Islam Khan, Medical Officer, Clinical Neuro-Surgery, National Institute of Neuro-Sciences and Hospital Sher-E-Bangla

Nagar, Dhaka-1207, Bangladesh. Mobile: +880 1713 455 662, E-mail: [email protected]

Page |

103

Left common carotid artery. The left brachiocephalic vein joins with the right brachiocephalic vein to form the superior vena cava on the right side of the body. Therefore, the left brachiocephalic vein must course across the mediastinum to reach its destination. The left subclavian artery and vein are lateral to the mediastinum, while the left jugular and common carotid travel vertically. 74. A 66-year-old man undergoes a left lung lobectomy for a tumour originating from the lingula. Which lobe of the left lung contains the lingula? Select one answer only.

Inferior lobe« YOUR ANSWER

Middle lobe

Superior lobe« CORRECT ANSWER

Apical lobe

Basal lobe. The left lung is divided into superior and inferior lobes by a long deep oblique fissure. The superior lobe has a wide cardiac notch on its anterior border. The anteroinferior part of the superior lobe also has a small tongue-like projection called the lingula. 75. A 65-year-old heavy smoker with type 2 diabetes mellitus presents to A and E with central chest pain radiating to his back. Whilst in resus, he is noted to drop his blood pressure, and on careful examination his JVP is increases on inspiration and his pulse fades. What is the most likely complication of aortic dissection that has developed? Select one answer only.

Aortic regurgitation« YOUR ANSWER

Cardiac tamponade« CORRECT ANSWER

Myocardial ischaemia

Myocardial infarction

Right ventricular failure. Aortic dissection results from an aortic intimal tear with blood tracking between the intima and

media to create a false lumen. In doing this the aortic wall is weakened and may rupture.

Intrapericardial rupture will cause cardiac tamponade. Additionally, as the dissection progresses,

flow down arterial branches may be compromised. Thus myocardial, cerebral and limb ischaemia

are potential consequences. Lastly, aortic dissection can lead to dilatation of the aortic root and

annulus leading to acute aortic regurgitation.

76. A 72-year-old diabetic male presents with central chest pain radiating to his neck. His ECG shows he has developed complete heart block thought to be secondary to ischaemia affecting the AV node. In what proportion of people is the AV node supplied by the right coronary artery? Select one answer only.

1%« YOUR ANSWER

MyPasTest: MRCS A Online - Jan Exam 2015 02. Anatomy; Thorax(120Qs)

-----------------------------------------------------------------------------------------------------------------

------------------------------------------------------------------------------------------------------------------------------------------------------------------------- Dr Mohammed Shamsul Islam Khan, Medical Officer, Clinical Neuro-Surgery, National Institute of Neuro-Sciences and Hospital Sher-E-Bangla

Nagar, Dhaka-1207, Bangladesh. Mobile: +880 1713 455 662, E-mail: [email protected]

Page |

104

25%

50%

80%« CORRECT ANSWER

95%. The right coronary artery gives off a posterior interventricular branch. The right coronary artery

supplies the AV node in 80% of cases.

77. An 82-year old man undergoes insertion of central line through the right internal jugular vein. The wire used during the procedure damages the right atrium. Which of the following is true regarding the right atrium? Select one answer only.

It contains the sinoatrial node« YOUR ANSWER (Correct)

It forms the inferior border of the heart

It has an anterior wall formed by the interatrial septum

It has the coronary sinus opening above the septal cusp of the mitral valve

It lies posterior to the left atrium. The right atrium forms the right border of the heart, lies anterior to the left atrium and so its posterior wall is the interatrial septum. The sinoatrial node lies near the opening of the superior vena cava, lateral to the sulcus limitans. The coronary sinus (responsible for the venous drainage of the myocardium) opens into the atrium above both the opening of the inferior vena cava and the septal cusp of the tricuspid valve. 78. A 6-year-old boy accidentally aspirates a small coin. A chest X-ray shows this to be lodged in the right main bronchus. Which of the following descriptions of the main bronchial airways is correct? Select one answer only.

Aspiration pneumonitis is more common in the right lower lobe than the left« YOUR ANSWER (Correct)

The azygous vein crosses the left main bronchus

The left bronchus is shorter than the right

The left main bronchus divides before entering the lung

The right main bronchus has a narrower diameter than the left. The right main bronchus is shorter (approximately 2.5 cm long), wider and runs more vertically than the left main bronchus. The right main bronchus gives off the lobe branch (before entering the lung hilum) and passes inferior to the pulmonary artery before entering the hilum of the lung (approximately T5). It is important to remember that the azygos vein arches over the right main bronchus from the posterior aspect as it passes to the SVC. The pulmonary artery lies inferior and then anterior to it.

MyPasTest: MRCS A Online - Jan Exam 2015 02. Anatomy; Thorax(120Qs)

-----------------------------------------------------------------------------------------------------------------

------------------------------------------------------------------------------------------------------------------------------------------------------------------------- Dr Mohammed Shamsul Islam Khan, Medical Officer, Clinical Neuro-Surgery, National Institute of Neuro-Sciences and Hospital Sher-E-Bangla

Nagar, Dhaka-1207, Bangladesh. Mobile: +880 1713 455 662, E-mail: [email protected]

Page |

105

The left main bronchus is about 5 cm long and, unlike the right, does not give off any branches before entering the hilum of the left lung at the level of T6. 79. A patient with a cystic swelling in his left chest underwent a computer tomography-guided biopsy. The radiologist inserted the biopsy needle into the ninth intercostal space along the mid-axillary line to aspirate the swelling and obtain tissue for histological diagnosis. The swelling is most likely to be in which space? Single best answer question – choose ONE true option only.

Cardiac notch« YOUR ANSWER

Costodiaphragmatic recess« CORRECT ANSWER

Costomediastinal recess

Cupola

Oblique pericardial sinus. The costodiaphragmatic recess is the lowest extent of the pleural cavity or sac. It is the part of the pleural sac where the costal pleura changes into the diaphragmatic pleura. It is also the area into which a needle is inserted for thoracocentesis and it is found at different levels in different areas of the thorax. At the mid-clavicular line, the costodiaphragmatic recess is between ribs 6 and 8; at the mid-axillary line it is between ribs 8 and 10; and at the paravertebral line it is between ribs 10 and 12. Therefore, inserting the needle just below the ninth rib at the mid-axillary line should put you in the costodiaphragmatic recess. The cardiac notch is a structure on the left lung, which separates the lingula below from the upper portion of the superior lobe of the left lung. The costomediastinal recess is found where the costal pleura become the mediastinal pleura. The cupola is the part of the pleural cavity, which extends above the level of the first rib into the root of the neck. The oblique pericardial sinus is an area of the pericardial cavity located behind the left atrium of the heart. 80. You suspect a lower respiratory tract infection in an elderly post operative patient and request a chest X-ray. Which of the following chest X-ray appearances is most likely? Single best answer question – choose ONE true option only.

Blunting of the costophrenic angles« YOUR ANSWER

Indistinct hemidiaphragms« CORRECT ANSWER

Kerley B lines

Indistinct right heart border

Indistinct left heart border. Blunting of the costophrenic angles occurs when a (relatively) small volume of fluid has collected

in the pleural spaces. This can occur in a number of scenarios, but it is not the most likely

appearance in the above patient. Immobilisation puts such a patient at risk of basal atelectasis,

subsequent consolidation and therefore a less distinct air-tissue interface at the diaphragm on

chest X-ray.

MyPasTest: MRCS A Online - Jan Exam 2015 02. Anatomy; Thorax(120Qs)

-----------------------------------------------------------------------------------------------------------------

------------------------------------------------------------------------------------------------------------------------------------------------------------------------- Dr Mohammed Shamsul Islam Khan, Medical Officer, Clinical Neuro-Surgery, National Institute of Neuro-Sciences and Hospital Sher-E-Bangla

Nagar, Dhaka-1207, Bangladesh. Mobile: +880 1713 455 662, E-mail: [email protected]

Page |

106

Kerley B lines occur when there is interstitial oedema. An indistinct right heart border occurs with

right middle lobe consolidation, and an indistinct left heart border occurs when there is left upper

lobe consolidation. Both of these are less likely sites for consolidation given the above history.

81. A 72-year-old hypertensive male undergoes a CABG following an angiogram whilst being investigated for angina, which showed triple vessel disease. Which of the following should be prescribed long term to reduce further myocardial complications or development of atherosclerosis in the bypass conduits? Select one answer only.

Abciximab« YOUR ANSWER

Aspirin« CORRECT ANSWER

Dipyridamole

Enoxaparin

Warfarin. The overall mortality for CABG is about 5–8% at 5 years. As part of the long-term prevention of further myocardial complications and development of atherosclerosis in the bypass conduits or in the native arteries, lifelong aspirin is advocated. Warfarin is not required following CABG unless a mechanical valve has been inserted or for persistent postoperative atrial fibrillation. Coronary artery stenting is a useful technique for recurrent coronary artery disease. Dipryridamole is used mainly for the secondary prevention of stroke and TIA. Interestingly it can cause a vasodilation of healthy coronary arteries, and has no effect on narrowed coronary arteries. This can result in a „steal‟ phenomena occurring which will worsen ischaemic areas. 82. A 67-year-old male presents with severe chest pain following a bout of prolonged vomiting. Computerised tomography reveals oesophageal perforation with associated mediastinitis. Which artery supplies the upper one third of the oesophagus?

Ascending pharyngeal artery« YOUR ANSWER

Inferior thyroid artery« CORRECT ANSWER

Lingual artery

Oesophageal artery

Superior thyroid artery. The oesophagus is divided into three anatomical sections. The upper one third receives blood supply from the inferior thyroid artery, the second from the oesophageal arteries, and the third from the left gastric artery. 83. Where would you visualise the azygous lobe on an antero-posterior (A-P) chest X-ray? Single

best answer - choose ONE true option only.

Left lower zone« YOUR ANSWER

Left middle zone

MyPasTest: MRCS A Online - Jan Exam 2015 02. Anatomy; Thorax(120Qs)

-----------------------------------------------------------------------------------------------------------------

------------------------------------------------------------------------------------------------------------------------------------------------------------------------- Dr Mohammed Shamsul Islam Khan, Medical Officer, Clinical Neuro-Surgery, National Institute of Neuro-Sciences and Hospital Sher-E-Bangla

Nagar, Dhaka-1207, Bangladesh. Mobile: +880 1713 455 662, E-mail: [email protected]

Page |

107

Left upper zone

Right upper zone« CORRECT ANSWER

Right lower zone. An azygous lobe is seen in about 0.5% of routine chest X-rays and is a normal variant. It is seen as

a „reverse comma sign‟ behind the medial end of the right clavicle.

84. A 2-day-old boy presents with regurgitation of saliva, choking after feeds and drooling. A diagnosis of oesophageal atresia without fistula is made following further investigation. Which foetal abnormality would you have expected to see during antenatal ultrasound screening?

Anencephaly« YOUR ANSWER

Aneuploidy

Oligohydramnios

Polyhydramnios« CORRECT ANSWER

Subchorionic haemorrhage. Oesophageal atresia occurs in 1 in every 3,000-4,500 live births. In cases of atresia without fistulae, nearly all mothers will have demonstrable polyhydramnios on ultrasound screening. 85. Endoderm is one of the germ layers formed during embryogenesis. Which of the following organs is a derivative of the endoderm? Single best answer question – choose ONE true option only.

Adrenal medulla« YOUR ANSWER

Dermis of the skin

Epithelial part of the tympanic cavity« CORRECT ANSWER

Gonads

Lens. Derivatives of endoderm include the epithelium of the gastrointestinal tract and its associated glands as well as glandular cells of the liver and pancreas, epithelium of the urachus and urinary bladder, epithelium of respiratory passages (the pharynx, trachea, bronchi and alveoli), epithelial parts of the tonsils, thyroid, parathyroids, tympanic cavity and thymus and epithelial parts of the anterior pituitary. 86. In this case, which region of myocardium would you expect to be most affected? Single best

answer - choose ONE true option only.

The right atrium« YOUR ANSWER

The right ventricle

The anterior septum

The anterior left ventricular wall

MyPasTest: MRCS A Online - Jan Exam 2015 02. Anatomy; Thorax(120Qs)

-----------------------------------------------------------------------------------------------------------------

------------------------------------------------------------------------------------------------------------------------------------------------------------------------- Dr Mohammed Shamsul Islam Khan, Medical Officer, Clinical Neuro-Surgery, National Institute of Neuro-Sciences and Hospital Sher-E-Bangla

Nagar, Dhaka-1207, Bangladesh. Mobile: +880 1713 455 662, E-mail: [email protected]

Page |

108

The posterior portion interventricular septum and the posterior left ventricular wall« CORRECT ANSWER.

The coronary system consists of left and right coronary arteries, which arise immediately above the

aortic valve. They are unique in that they fill during diastole, when not occluded by valve cusps and

when not squeezed by myocardial contraction.

The right coronary artery arises from the right coronary sinus, giving off branches supplying the

right atrium and right ventricle. It then continues as the posterior descending coronary artery,

which supplies the posterior portion of the interventricular septum and the posterior left ventricular

wall.

The left coronary artery divides into the left anterior descending (LAD) and circumflex arteries. The

LAD runs in the anterior interventricular groove and supplies the anterior septum and the anterior

left ventricular wall. The left circumflex artery gives off branches that supply the left atrium and left

ventricle.

The sinus node is supplied by the right coronary artery in around 60% of people, the AV node in

around 90%.

87. A 29-year-old woman undergoes an emergency thoracotomy following a gunshot wound to the left side of the chest. On exposing the left main bronchus, which of the following structures may be encountered on its anterior surface? Select one answer only.

Aorta« YOUR ANSWER

Azygos vein

Phrenic nerve« CORRECT ANSWER

Recurrent laryngeal nerve

Vagus nerve. The left main bronchus passes downwards and outwards below the aortic arch, anterior to the descending aorta and oesophagus. The pulmonary artery loops over it, the vagus nerve lies just posterior, the phrenic nerve in front, and the hemiazygos vein is posterior to the aorta. 88. A hypertensive, heavy smoking, 73-year-old man suffers a massive cardiac infarct following

occlusion of his anterior interventricular artery, (anterior descending artery). Angiography is

performed to demonstrate the coronary vessels. Which anatomical relationship of these vessels

should be borne in mind? Single best answer question – choose ONE true option only.

The anterior interventricular artery arises above the left posterior aortic cusp« YOUR ANSWER

The anterior interventricular artery supplies almost all of the left ventricle« CORRECT ANSWER

There is a rich collateral circulation between the right and left coronary arteries

The circumflex artery is the major branch of the right coronary artery

The posterior interventricular artery is a branch of the circumflex artery.

MyPasTest: MRCS A Online - Jan Exam 2015 02. Anatomy; Thorax(120Qs)

-----------------------------------------------------------------------------------------------------------------

------------------------------------------------------------------------------------------------------------------------------------------------------------------------- Dr Mohammed Shamsul Islam Khan, Medical Officer, Clinical Neuro-Surgery, National Institute of Neuro-Sciences and Hospital Sher-E-Bangla

Nagar, Dhaka-1207, Bangladesh. Mobile: +880 1713 455 662, E-mail: [email protected]

Page |

109

The left coronary artery arises above the left posterior aortic cusp. After a short course it divides

into an anterior interventricular and a circumflex branch. The former is the main arterial supply to

the left ventricle. Unfortunately, there is only a poor collateral supply between the branches of the

two coronary arteries. The posterior interventricular artery arises from the right coronary artery.

89. A 39-year-old woman undergoes a transthoracic echo following the recent onset of heart failure with cyanosis. This confirms an Atrial Septal Defect (ASD) and it is thought she has Eisenmenger syndrome. The foramen ovale normally closes at birth to form the oval fossa. Which of the following best describes the location of the oval fossa within the right atrium? Select one answer only.

Anterolateral wall« YOUR ANSWER

Anteromedial wall

Inferior aspect

Posterolateral wall

Posteromedial wall « CORRECT ANSWER. The two parts of the right atrium are separated externally by a groove on the posterior aspect of the

atrium known as the sulcus terminalis and internally by the crista terminalis, which extends

between the two vena caval orifices. The fossa ovalis is found on the interatrial septum, which

forms the posteromedial wall of the right atrium. The opening of the coronary sinus is guarded by a

semicircular valve that closes the orifice during contraction of the right atrium.

90. A 35-year-old scaffolder falls from a height of ten metres and complains of back pain. On examination, he is unable to flex his knee, but can extend it following passive flexion. At which vertebral level has there been an injury to the spinal cord?

L1« YOUR ANSWER

L3

L5« CORRECT ANSWER

S2

T12. Quadriceps (knee extenders) includes rectus femoris, vastus lateralis, vastus medialis and vastus intermedius. These muscles are innervated by fibres originating from the posterior division of ventral rami of the femoral nerve (L2, 3, 4). The hamstrings (biceps femoris, semitendinosus and semimembranosus) are supplied by tibial component of sciatic nerve (L5, S1) and flex the knee joint. 91. Which of the following structures is located in the anterior mediastinum on computed tomography (CT)? Single best answer question – choose ONE true option only.

Thymus« YOUR ANSWER

MyPasTest: MRCS A Online - Jan Exam 2015 02. Anatomy; Thorax(120Qs)

-----------------------------------------------------------------------------------------------------------------

------------------------------------------------------------------------------------------------------------------------------------------------------------------------- Dr Mohammed Shamsul Islam Khan, Medical Officer, Clinical Neuro-Surgery, National Institute of Neuro-Sciences and Hospital Sher-E-Bangla

Nagar, Dhaka-1207, Bangladesh. Mobile: +880 1713 455 662, E-mail: [email protected]

Page |

110

Oesophagus

Aorta

Heart

Trachea. The anterior mediastinum is bordered anteriorly by the sternum and posteriorly by the great vessels. It contains the thymus, lymph nodes, fat, and vessels. Disorders of the anterior mediastinum are generally thymic, thyroid (substernal goitre), teratoma (and other germ cell tumors), and lymphomas (Hodgkin's disease, non-Hodgkin's lymphoma). 92. A 43-year-old woman undergoes parathyroid surgery. Immediately following the surgery she is noted to have developed a hoarse voice. Which of the following is the most accurate description of the recurrent laryngeal nerve? Select one answer only.

It lies deep to the inferior thyroid artery« YOUR ANSWER

It runs between the oesophagus and trachea in the neck« CORRECT ANSWER

It should be retracted during tracheostomy to avoid damage

It supplies the cricothyroid muscle

It supplies the mucous surface of the vocal cords. The recurrent laryngeal nerve is branch of the vagus nerve. It supplies all the intrinsic muscles of the larynx except the cricothyroid and is sensory inferior to the vocal folds. In the neck the recurrent laryngeal nerves on both sides follow the same course, ascending in the tracheo-oesophageal groove. As the nerve passes the lateral lobe of the thyroid it is closely related to the inferior thyroid artery. The superior laryngeal nerve supplies the vocal cord mucosa. Damage to one recurrent laryngeal nerve leads to a hoarse voice. Damage to both nerves can lead to aphonia. 93. A 56-year-old male presents to A&E with severe vomiting and chest discomfort. His chest X-ray shows air in the mediastinum. Which of the following disease processes may cause this appearance? Select one answer only.

Aortic rupture« YOUR ANSWER

Aortic dissection

Cardiac tamponade

Oesophageal perforation« CORRECT ANSWER

Pericarditis. Pneumomediastinum is the presence of air in the mediastinal tissues and can be readily seen on a chest radiograph. It is a hallmark of oesophageal perforation and large airway (trachea or bronchus) injury, and therefore must be taken seriously. Air may track into the mediastinum via the diaphragm following perforation of an intra-abdominal viscus. Isolated aortic rupture may cause mediastinal widening, but will not present as pneumomediastinum. Similarly, pericarditis does not cause pneumomediastinum.

MyPasTest: MRCS A Online - Jan Exam 2015 02. Anatomy; Thorax(120Qs)

-----------------------------------------------------------------------------------------------------------------

------------------------------------------------------------------------------------------------------------------------------------------------------------------------- Dr Mohammed Shamsul Islam Khan, Medical Officer, Clinical Neuro-Surgery, National Institute of Neuro-Sciences and Hospital Sher-E-Bangla

Nagar, Dhaka-1207, Bangladesh. Mobile: +880 1713 455 662, E-mail: [email protected]

Page |

111

94. A 19-year old woman is shot in the chest at the level of T5. Which of the following structures is most likely to be injured at this level? Select one answer only.

The thoracic duct as it crosses the midline« YOUR ANSWER (Correct)

The bifurcation of the trachea (the carina)

The commencement of the aortic arch

The junction of the azygous vein as it enters the superior vena cava

T4 vertebral body. The thoracic duct crosses the midline at T5. The manubriosternal joint corresponds to the T4 vertebral level. At this level the following structures are found: - bifurcation of the trachea - start of the aortic arch - azygous vein entering the superior vena cava - the left recurrent laryngeal nerve loops round the ligamentum arteriosum. 95. The aortic arch and the junction of the azygos vein with the superior vena cava are positioned

on the same plane as which spinal column structure?

T3 vertebrae« YOUR ANSWER

T4 vertebrae

T5 vertebrae

T3/4 intervertebral disc

T4/5 intervertebral disc« CORRECT ANSWER. The manubriosternal joint marks the thoracic plane, which is at the level of the T4/5 intervertebral disc. Other structures found on this plane include the aortic arch, the junction of the azygos vein with the SVC, the 2nd costal cartilages, the tracheal bifurcation, the ligamentum arteriosum and the decussation of the thoracic duct. The manubriosternal joint is also known as the Angle of Louis. 96. A 45-year-old man is undergoing decompression of thoracic outlet syndrome. During this, the thoracic duct is thought to be injured and a drain is placed. Where does the thoracic duct normally drain into? Select one answer only.

Azygos vein« YOUR ANSWER

Left brachiocephalic vein« CORRECT ANSWER

Left external jugular vein

Right brachiocephalic vein

Superior vena cava.

MyPasTest: MRCS A Online - Jan Exam 2015 02. Anatomy; Thorax(120Qs)

-----------------------------------------------------------------------------------------------------------------

------------------------------------------------------------------------------------------------------------------------------------------------------------------------- Dr Mohammed Shamsul Islam Khan, Medical Officer, Clinical Neuro-Surgery, National Institute of Neuro-Sciences and Hospital Sher-E-Bangla

Nagar, Dhaka-1207, Bangladesh. Mobile: +880 1713 455 662, E-mail: [email protected]

Page |

112

The left brachiocephalic vein drains blood from: the cervical vertebrae via both vertebral veins; the

thyroid gland by the inferior thyroid veins; the first left intercostal space via the left superior

intercostal veins; and all the anterior intercostal spaces by the anterior intercostal veins draining

into the internal thoracic veins. The thoracic duct enters the vein at its commencement behind the

left sternoclavicular joint. The bronchial veins drain into the azygos/hemiazygos systems.

97. An SHO has been asked to aspirate some pleural fluid for culture and sensitivity from the left pleural space of a 65-year-old man who has postpneumonic effusion. If the SHO wants to aspirate the fluid with the patient sitting up in bed, where would the fluid tend to accumulate? Single best answer question – choose ONE true option only.

Costodiaphragmatic recess« YOUR ANSWER (Correct)

Costomediastinal recess

Cupola

Hilar reflection

Middle mediastinum. The costodiaphragmatic recess is the lowest extent of the pleural cavity or sac. It is the part of the pleural sac where the costal pleura changes into the diaphragmatic pleura. Because this is the inferior part of the pleural sac, fluid in the pleural sac will fall to this region when a patient sits up. The costodiaphragmatic recess is also the area into which a needle is inserted for thoracocentesis and it is found at different levels at different areas of the thorax. At the mid-clavicular line, the costodiaphragmatic recess is between ribs 6 and 8; at the mid-axillary line it is between ribs 8 and 10; and at the paravertebral line it is between ribs 10 and 12. The costomediastinal recess is found where the costal pleura become the mediastinal pleura. The cupola is the part of the pleural cavity that extends above the level of the first rib into the root of the neck. The hilar reflection is the point at the root of the lung where the mediastinal pleura is reflected and becomes continuous with the visceral pleura. Finally, the middle mediastinum is the space in the mediastinum that contains the heart, pericardium, great vessels and bronchi (at the roots of the lung). 98. The human embryonic heart rate is nearly twice that of the adult. Which of the following statements regarding the development of the human heart is CORRECT? Single best answer question – choose ONE true option only.

In the second week, the endocardial tubes begin to fuse to form a single tube« YOUR ANSWER

The septum primum appears in the eighth week

The heart begins to beat in the fourth week« CORRECT ANSWER

The primordium of the heart forms in the cardiogenic plate located at the caudal end of the embryo

The bulboventricular loop is formed in the sixth week. The primordium of the heart forms in the cardiogenic plate located at the cranial end of the embryo. Angiogenic cell clusters, which lie in a horseshoe-shaped configuration in the plate, coalesce to

MyPasTest: MRCS A Online - Jan Exam 2015 02. Anatomy; Thorax(120Qs)

-----------------------------------------------------------------------------------------------------------------

------------------------------------------------------------------------------------------------------------------------------------------------------------------------- Dr Mohammed Shamsul Islam Khan, Medical Officer, Clinical Neuro-Surgery, National Institute of Neuro-Sciences and Hospital Sher-E-Bangla

Nagar, Dhaka-1207, Bangladesh. Mobile: +880 1713 455 662, E-mail: [email protected]

Page |

113

form two endocardial tubes. These tubes are then forced into the thoracic region due to cephalic and lateral foldings, where they fuse together forming a single endocardial tube during the third week. The heart begins to beat in the fourth week at about the same time that the septum primum appears and the bulboventricular loop is formed. From the fourth week onwards, septa begin to grow in the atria, ventricle and bulbus cordis to form right and left atria, right and left ventricles and the two great vessels - the pulmonary artery and the aorta. By the end of the eighth week, partitioning is completed and the fetal heart has formed. 99. A 46-year old man is found to have a suspicious lesion in the left lower lung lobe on a CT scan. He undergoes bronchoscopy in order to obtain a histological sample. Which of the following statements best describes the anatomy of the left lung? Select one answer only.

The left lung has ten bronchopulmonary segments« YOUR ANSWER (CORRECT)

The left lung has three bronchial openings

The left lung has three pulmonary veins

The left lung has three lobes

The left lung receives its an arterial supply from the left pulmonary artery. The left lung has two lobes, in contrast to the right, which has three lobes. The left lung has two bronchial openings and ten bronchopulmonary segments. Two pulmonary veins arise from the left lung, one above and one below the oblique fissure. Note that the arterial supply to the lung parenchyma itself originates from the bronchial arteries which arise from the aorta. The pulmonary arteries and veins transport deoxygenated and oxygenated blood from and to the heart, respectively. They are not the primary arterial supply of the lung parenchyma. 100. An 87-year-old female recovering from a stroke is noted to be increasingly short of breath and confused. On examination, anteriorly her chest is clear but posteriorly harsh bronchial breathing is detected in the lower and mid zone bilaterally. Pneumonia is diagnosed. Your consultant questions you about the anatomy of the lungs. Select one correct answer only?

The right lung has two lobes separated by oblique fissure« YOUR ANSWER

The right lung has three lobes separated by oblique and horizontal fissures« CORRECT ANSWER

Regarding the lung roots, the pulmonary artery lies superiorly and the pulmonary vein lies inferiorly whilst the bronchus lies anteriorly

The anterior margin of the left lung is straight, unlike that of the right lung

The lung is composed of approximately 300 thousand alveoli each around 0.3mm in diameter. The right lung has three lobes separated by oblique and horizontal fissures, whilst the left lung has two, separated by oblique fissure. Lung roots – pulmonary artery lies superiorly. Bronchus lies posteriorly and Pulmonary veins lie inferiorly.

MyPasTest: MRCS A Online - Jan Exam 2015 02. Anatomy; Thorax(120Qs)

-----------------------------------------------------------------------------------------------------------------

------------------------------------------------------------------------------------------------------------------------------------------------------------------------- Dr Mohammed Shamsul Islam Khan, Medical Officer, Clinical Neuro-Surgery, National Institute of Neuro-Sciences and Hospital Sher-E-Bangla

Nagar, Dhaka-1207, Bangladesh. Mobile: +880 1713 455 662, E-mail: [email protected]

Page |

114

The right lung is larger and heavier than the left but it is also shorter and wider because the right dome of the diaphragm is higher and the heart bulge more to the left. The anterior margin of the left lobe has the cardiac notch. The human lung is composed of approximately 300 million alveoli each around 0.3mm in diameter. 101. A 58-year-old man undergoes coronary angioplasty for unstable angina. He suffers an iatrogenic rupture of the right coronary artery at the time of angioplasty. Which of the following statements is true regarding the right coronary artery? Select one answer only.

It does NOT supply the left atrium« YOUR ANSWER

It overlies the right atrial appendage

It lies posterior to the infundibulum of the right ventricle

It originates in the posterior aortic sinus

It supplies the sinoatrial node« CORRECT ANSWER. The right coronary artery originates from the anterior aortic sinus of the aortic root. It supplies the right atrium and ventricle and variable amounts of the left atrium and ventricle. It supplies the sinoatrial node. The sinoatrial node artery passes backwards between the right auricle and aorta, and forms a vascular ring around the termination of the superior vena cava. Arteriolar anastomoses between the terminations of the right and left coronary arteries exist, but are too few and small in calibre to compensate significantly in acute coronary artery occlusion. Iatrogenic coronary artery rupture may require emergency placement of a pericardial drain to avoid pericardial tamponade. 102. A CT 2 in cardiothoracic surgery performs his first midline sternotomy. During this heavy bleeding is noted. Which of the following structures is most likely to be damaged? Select one answer only.

Descending aorta« YOUR ANSWER

Inferior vena cava

Internal thoracic artery

Right brachiocephalic vein« CORRECT ANSWER

Superior epigastric artery. Sternotomy is done to gain access to the heart, lung or the mediastinal structures. In a midline sternotomy incision the following structures are likely to be affected: the brachiocephalic trunk, the left common carotid artery, the left subclavian artery, both brachiocephalic veins, trachea, oesophagus, phrenic nerves, both vagi, thoracic duct, left recurrent laryngeal nerve and thymus in children. The internal thoracic or the internal mammary arteries run 2 cm lateral to the lateral edge of the sternal border and hence not affected in a midline sternotomy incision. The inferior vena cava enters the thoracic cavity piercing the diaphragm at the level of T8 vertebra and opens into the base

MyPasTest: MRCS A Online - Jan Exam 2015 02. Anatomy; Thorax(120Qs)

-----------------------------------------------------------------------------------------------------------------

------------------------------------------------------------------------------------------------------------------------------------------------------------------------- Dr Mohammed Shamsul Islam Khan, Medical Officer, Clinical Neuro-Surgery, National Institute of Neuro-Sciences and Hospital Sher-E-Bangla

Nagar, Dhaka-1207, Bangladesh. Mobile: +880 1713 455 662, E-mail: [email protected]

Page |

115

of the right atrium. It is not contained within the mediastinum and so not affected during a midline sternotomy incision.. 103. A 60-year-old previously fit and well man dies suddenly after two separate syncopal episodes. At post mortem examination, which one of the following congenital cardiac defects that could be associated with his sudden death would you expect to find? Single best answer question – choose ONE true option only.

Dextrocardia with situs inversus« YOUR ANSWER

Patent foramen ovale

Bicuspid aortic valve« CORRECT ANSWER

Ventricular septal defect

Single coronary artery. Whilst isolated dextrocardia can be associated with severe cardiac anomalies, dextrocardia with

situs inversus has a low incidence of accompanying defects and the heart functions normally.

Isolated patent foramen ovale is of no haemodynamic significance. The relatively common (1-2% of

people) bicuspid aortic valve leads to accelerated calcific stenosis, which is associated with

syncope and sudden death. Small ventricular septal defects are insignificant, whereas large defects

lead to massive left to right shunts. A single coronary artery is not common, but does occur. When

present, there is no clinical consequence unless disease (such as atheroma) affects the vessel.

104. A 23-year-old female is being considered for a thoracoscopic sympathectomy for refractory hyperhidrosis affecting both axillae and hands. She has failed to respond to conservative and medical therapy to date. Which of the following descriptions most accurately describes how the sympathetic trunk passes from the abdomen to the thorax? Select one answer only.

Adjacent to the inferior vena cava and right phrenic nerve« YOUR ANSWER

Adjacent to the oesophagus within the right crus of the diaphragm

Posterior to the lateral arcuate ligament over quadratuslumborum

Posterior to the medial arcuate ligament over psoas major« CORRECT ANSWER

Within the vertebral canal. The thoracic sympathetic chain lies on the heads of the ribs, anterior to the posterior intercostal

vessels, immediately under cover of the pleura, with the splanchnic nerves passing from the chain

medially and anteriorly over the vertebral bodies. The thoracic sympathetic chain receives white

rami from all the intercostal nerves, and passes into the abdomen under the medial arcuate

ligament of the diaphragm.

105. A 72-year-old male presents with tearing chest pain radiating to his back. He has a past medical history of poorly controlled hypertension and is a heavy smoker. Imaging of his thorax shows a Stanford type B aortic dissection. How is this type of dissection normally managed? Select one answer only.

MyPasTest: MRCS A Online - Jan Exam 2015 02. Anatomy; Thorax(120Qs)

-----------------------------------------------------------------------------------------------------------------

------------------------------------------------------------------------------------------------------------------------------------------------------------------------- Dr Mohammed Shamsul Islam Khan, Medical Officer, Clinical Neuro-Surgery, National Institute of Neuro-Sciences and Hospital Sher-E-Bangla

Nagar, Dhaka-1207, Bangladesh. Mobile: +880 1713 455 662, E-mail: [email protected]

Page |

116

Endovascular repair« YOUR ANSWER

Hybrid procedure

Lifestyle advice

Medical treatment« CORRECT ANSWER

Open surgery. Aortic dissection results from an aortic intimal tear with blood tracking between the intima and

media to create a false lumen. In doing this the aortic wall is weakened and may rupture. Stanford

type A dissections involve the ascending aorta or aortic arch and are treated surgically. Stanford

type B dissections do not involve the arch or ascending aorta and are treated medically.

There is an association with Marfan‟s syndrome. The operation involves replacing the ascending

aorta with a prosthetic graft to prevent dissection back towards the aortic valve and the coronary

ostia. The rest of the aorta is not replaced. Aortic dissection is normally seen as widened

mediastinum on a chest radiograph, but a normal chest radiograph does not exclude aortic

dissection.

106. A 45-year-old female is brought in following a trauma call, having been involved in a RTA. On arrival she is very short of breath, and complaining of right chest pain. On examination, she is cyanosed, has absent air entry on the right chest and is hyper-resonant on percussion. The trachea is deviated from the mid-line. What is the immediate management? Select one answer only.

Chest drain insertion on right« YOUR ANSWER

Chest CT

Chest X-ray

Needle thoracocentesis« CORRECT ANSWER

Thoracotomy. A tension pneumothorax results from blunt or penetrating trauma, with or without associated rib fractures. A tension pneumothorax results from any lung parenchymal or bronchial injury that acts as a one-way valve and allows atmospheric air to move into an intact pleural space but prevents the exit of that air. As pressure within the intrapleural space increases, there is deviation of the trachea, heart and mediastinal structures to the contralateral side. The other signs and symptoms include: chest pain, dyspnoea or respiratory distress, cyanosis, decreased breath sounds on the affected side, hyperresonance of the chest wall on percussion of the affected side, distended jugular veins (due to obstruction of superior vena cava) and hypotension. Tension pneumothorax is a clinical diagnosis and a life-threatening emergency. It is essential to relieve the pressure as soon as possible and no time should be wasted in the Emergency Department to acquire radiological imaging once a diagnosis of tension pneumothorax is made.

MyPasTest: MRCS A Online - Jan Exam 2015 02. Anatomy; Thorax(120Qs)

-----------------------------------------------------------------------------------------------------------------

------------------------------------------------------------------------------------------------------------------------------------------------------------------------- Dr Mohammed Shamsul Islam Khan, Medical Officer, Clinical Neuro-Surgery, National Institute of Neuro-Sciences and Hospital Sher-E-Bangla

Nagar, Dhaka-1207, Bangladesh. Mobile: +880 1713 455 662, E-mail: [email protected]

Page |

117

The immediate management consists of inserting a large-bore cannula (14G or 16G) (needle thoracocentesis) through the second intercostal space in the mid-clavicular line on the affected side; the needle should be over the top of the rib since the intercostal vessels and nerve run just below the rib. Although this is the ideal site of insertion and highly recommended, if required, the needle could be inserted through an intercostal space anywhere on the affected side. The diagnosis is confirmed when there is a sudden „rush‟ of air during needle thoracocentesis. In all patients, this should be followed by a formal chest drain (tube thoracostomy) once the patient is adequately resuscitated and stabilised. Tube thoracostomy is essential to prevent reaccumulation of air within the pleural cavity. 107. A 78-year-old man with pseudobulbar palsy, lying supine in bed, aspirates one of his tablets into his lungs while swallowing. It would be most likely to end up in which of the following bronchopulmonary segments? Single best answer question – choose ONE true option only.

Anterior segmental bronchus of the right superior lobe« YOUR ANSWER

Medial segmental bronchus of the right middle lobe

Superior segmental bronchus of the right inferior lobe« CORRECT ANSWER

Medial basal segmental bronchus of the left inferior lobe

Inferior segmental bronchus of the lingular. Remember: inhaled material tends to go into the right bronchus because it is bigger and more vertically orientated than the left! The superior segmental bronchus branches posteriorly off the intermediate bronchus or the inferior lobe bronchus, so it is the segmental bronchus most likely to receive the foreign bodies that enter the right main bronchus. 108. A 1-day-old child is noted to be in respiratory distress. Imaging reveals herniation of abdominal contents through the posterior aspect of the left hemidiaphragm. Which type of hernia does the child have?

Amyand’s hernia« YOUR ANSWER

Bochdalek’s hernia« CORRECT ANSWER

Hiatus hernia

Littre’s hernia

Morgagni’s hernia. Bochdalek and Morgagni herniae are both types of congenital diaphragmatic herniae. The former always occurs posteriorly (usually on the left side) and the latter anteriorly. Bochdalek herniae tend to be associated with more serious complications than Morgagni herniae, and have a much higher incidence. Urgent gastroenteric decompression and endotracheal intubation is required to stabilise the patient prior to surgery. The mnemonic „5 B‟s‟ can be used to recall differences between the two (B = Bochdalek, big, bad, baby, back). Littre‟s and Amyand‟s hernia are inguinal herniae that involve a Meckel‟s diverticulum and an inflamed appendix, respectively.

MyPasTest: MRCS A Online - Jan Exam 2015 02. Anatomy; Thorax(120Qs)

-----------------------------------------------------------------------------------------------------------------

------------------------------------------------------------------------------------------------------------------------------------------------------------------------- Dr Mohammed Shamsul Islam Khan, Medical Officer, Clinical Neuro-Surgery, National Institute of Neuro-Sciences and Hospital Sher-E-Bangla

Nagar, Dhaka-1207, Bangladesh. Mobile: +880 1713 455 662, E-mail: [email protected]

Page |

118

109. You are asked to assist your consultant with a pulmonary resection. During the procedure she begins to dissect tissue posterior to the right main bronchus. Which vessel is at risk of injury?

Azygos vein« YOUR ANSWER (Correct)

Hemiazygos vein

Superior vena cava

Thoracic aorta

Thoracic duct. The azygos vein arches posteriorly over the right main bronchus to join the superior vena cava adjacent to the root of the right lung. Care must also be taken to spare the vagus nerve, which lies posterior to the right main bronchus. 110. Regarding the muscles of respiration, which one of the following statements is true? Single best answer - select one answer only.

Quiet inspiration is predominantly due to the action of the diaphragm« YOUR ANSWER (Correct)

The scalene muscles play an important role in quiet expiration

Active inspiration is caused by the internal intercostal muscles

Internal oblique muscles alone are important in active expiration

The external intercostal muscles pull the ribs medially and inferiorly during active expiration. The predominant muscle of inspiration during quiet breathing is the diaphragm, a dome-shaped musculofibrous septum separating the thorax from the abdominal cavity. As the diaphragm contracts, pleural pressure drops, which lowers the alveolar pressure. This draws air into the lungs due to the pressure gradient from the mouth to the alveoli. Expiration during quiet breathing is a passive process. This is caused by the relaxation of the respiratory muscles and the return of the elastic lung and chest wall to their normal resting volume. During exertion or activity, the external intercostals help in inspiration by raising the lower ribs superiorly and laterally, so increasing the lateral and antero-posterior diameters of the thoracic cavity. The scalene muscles and sternocleidomastoids also help by raising and pushing out the upper ribs and the sternum. Active expiration is helped by the contraction of the abdominal wall muscles (internal oblique, external oblique, transversus abdominus and rectus abdominus). It increases the intra-abdominal pressure, which pushes up the diaphragm, so raising the pleural pressure and drives the air out of the lungs. The internal intercostals also help in active expiration by decreasing the thoracic volume (by pulling down medially and inferiorly). 111. A 25-year-old man was stabbed in the right supraclavicular fossa. The knife punctured the portion of the parietal pleura that extends above the first rib. This portion of the parietal pleura is called the: Single best answer question – choose ONE true option only.

Costodiaphragmatic recess« YOUR ANSWER

MyPasTest: MRCS A Online - Jan Exam 2015 02. Anatomy; Thorax(120Qs)

-----------------------------------------------------------------------------------------------------------------

------------------------------------------------------------------------------------------------------------------------------------------------------------------------- Dr Mohammed Shamsul Islam Khan, Medical Officer, Clinical Neuro-Surgery, National Institute of Neuro-Sciences and Hospital Sher-E-Bangla

Nagar, Dhaka-1207, Bangladesh. Mobile: +880 1713 455 662, E-mail: [email protected]

Page |

119

Costomediastinal recess

Costocervical recess

Cupola« CORRECT ANSWER

Endothoracic fascia. The cupola is the cervical parietal pleuron, which extends slightly above the level of the first rib into the root of the neck. The costodiaphragmatic recess is the part of the pleural sac where the costal pleura changes into the diaphragmatic pleura. It is the lowest extent of the pleural sac. The costomediastinal recess is found where the costal pleura become the mediastinal pleura. The endothoracic fascia is connective tissue between the inner chest wall and the costal parietal pleura. The costocervical recess is a made-up term. 112. An 81-year-old male presents with progressive dysphagia and weight loss. An OGD reveals a

tumour proximal to the gastro-oesophageal junction and histology confirms this is an oesophageal

adenocarcinoma. Which of the following best describes where the oesophagus passes through the

diaphragm? Select one answer only.

Posterior to the medial arcuate ligament« YOUR ANSWER

Posterior to the lateral arcuate ligament

Through the central tendon

Through the muscular sling of the left crus

Through the muscular sling of the right crus« CORRECT ANSWER. The oesophagus is formed at the lower border of the cricoid cartilage. It is crossed anteriorly by the

left main bronchus, lies behind the left atrium and passes through the muscular part of the

diaphragm to the left of the central tendon, through the muscular sling of the right crus. It is

innervated, in part, by the recurrent laryngeal nerve, not the phrenic nerve.

113. A male jockey is thrown from his horse and sustains a spinal cord injury. He presents with difficulty breathing and ultrasound scan reveals a right hemidiaphragmatic paralysis. From which spinal roots does the affected nerve originate?

C2-3« YOUR ANSWER

C2-4

C3-5« CORRECT ANSWER

C6-8

C7-8. The phrenic nerve has been injured and its function and origin can be recalled by the mnemonic „C3,4,5 keeps the diaphragm alive‟.

MyPasTest: MRCS A Online - Jan Exam 2015 02. Anatomy; Thorax(120Qs)

-----------------------------------------------------------------------------------------------------------------

------------------------------------------------------------------------------------------------------------------------------------------------------------------------- Dr Mohammed Shamsul Islam Khan, Medical Officer, Clinical Neuro-Surgery, National Institute of Neuro-Sciences and Hospital Sher-E-Bangla

Nagar, Dhaka-1207, Bangladesh. Mobile: +880 1713 455 662, E-mail: [email protected]

Page |

120

114. A 78-year-old male undergoes an oesophagectomy for oesophageal adenocarcinoma. Dissection around the intrathoracic oesophagus is very difficult, with significant bleeding. Which of the following is a posterior relation of the intrathoracic oesophagus? Select one answer only.

Aortic arch« YOUR ANSWER

Azygos vein

Left common carotid artery

Hemiazygos vein« CORRECT ANSWER

Right common carotid artery. Relations include the right and left bronchi, the trachea, the vagus nerve, pleura, the aortic arch and descending aorta and the thoracic duct. Within the thorax the relations of the oesophagus are: Anterior: • Left common carotid artery • Trachea • Left main bronchus • Pericardium Posterior: • Thoracic vertebrae • Thoracic duct • Hemiazygos vein • The descending aorta below On the left: • Left subclavian artery • Aortic arch • Left vagus nerve and its recurrent laryngeal branch • Thoracic duct • Left pleura On the right: • Right pleura • Azygos vein. 115. You are asked to see a 45-year-old man who is haemodynamically compromised and plan to insert a right subclavian line. He has a body mass index (BMI) of 38, where is the correct position for central venous cannulation? Single best answer - choose ONE true option only.

1 cm under the mid-point of the clavicle and 0.5 cm laterally« YOUR ANSWER

2 cm under the mid-point of the clavicle and 1 cm laterally« CORRECT ANSWER

2.5 cm under the mid-point of the clavicle and 2 cm laterally

0.5 cm under the mid-point of the clavicle and 1 cm laterally

MyPasTest: MRCS A Online - Jan Exam 2015 02. Anatomy; Thorax(120Qs)

-----------------------------------------------------------------------------------------------------------------

------------------------------------------------------------------------------------------------------------------------------------------------------------------------- Dr Mohammed Shamsul Islam Khan, Medical Officer, Clinical Neuro-Surgery, National Institute of Neuro-Sciences and Hospital Sher-E-Bangla

Nagar, Dhaka-1207, Bangladesh. Mobile: +880 1713 455 662, E-mail: [email protected]

Page |

121

1 cm under the mid-point of the clavicle and 1 cm laterally. In obese patients, the standard position for right subclavian central venous cannulation is 2 cm under the mid-point of the clavicle and 1 cm laterally. In thin patients the standard position for insertion is 1 cm under the mid-point of the clavicle and 0.5 cm laterally. Jugular vein cannulation is now the preferred choice for central venous catheterisation, as insertion under ultrasound guidance is associated with a much lower rate of complications than subclavian insertion. The major hazard of the subclavian approach is arterial puncture, as the artery lies close to the vein. 116. A previously well 61-year-old male presents to A and E with a marked right upper limb

weakness and right facial droop. He is also noted to have a dysphasia. On taking a history from his

wife it emerges he is due to attend hospital the following day for an elective inguinal hernia repair.

He is normally on warfarin for a mechanical valve, which he stopped 6 days ago on the advice of his

GP. What complication has occurred? Select one answer only.

Amaurosis fugax« YOUR ANSWER

Embolic stroke« CORRECT ANSWER

Haemorrhagic stroke

Prosthetic endocarditis

Subdural haematoma. Mechanical valvular prostheses require lifelong anticoagulation to prevent emboli and valvular

obstruction with thrombus. Heparin may be used to anticoagulate should warfarin need to be

withheld for any reason. Warfarin is a teratogen, and thus mechanical valves should be avoided in

those women planning to have children.

Prosthetic valve endocarditis is notoriously difficult to treat and will often require redo valve

replacement. Thus antibiotic prophylaxis prior to instrumentation is very important in those

patients with prosthetic valves.

117. Following a difficult cervical rib excision, the surgeon is concerned about a possible injury to the pleura. The patient reports ipsilateral chest pain worse on inspiration, is noted to have reduced breath sounds and the trachea remains central. There is hyperresonance evident on percussion of the ipsilateral chest wall. Which of the following complications is most likely from the history? Select one answer only.

Chylothorax« YOUR ANSWER

Haemothorax

Pleural effusion

Pneumothorax« CORRECT ANSWER

Tension pneumothorax.

MyPasTest: MRCS A Online - Jan Exam 2015 02. Anatomy; Thorax(120Qs)

-----------------------------------------------------------------------------------------------------------------

------------------------------------------------------------------------------------------------------------------------------------------------------------------------- Dr Mohammed Shamsul Islam Khan, Medical Officer, Clinical Neuro-Surgery, National Institute of Neuro-Sciences and Hospital Sher-E-Bangla

Nagar, Dhaka-1207, Bangladesh. Mobile: +880 1713 455 662, E-mail: [email protected]

Page |

122

The pleura is a serous membrane which invests the lung itself (visceral pleura) and lines the cavity containing the lung (parietal pleura). These two parts of the membrane are continuous at the lung hilum and enclose a small space occupied by about 5–10ml of fluid produced mainly by the parietal layer. Due to the elasticity of the lungs, intrapleural pressure is usually negative during quiet inspiration and expiration, but during forced expiration contraction of expiratory muscles raises the pressures in the pleural spaces and within the lungs and bronchi. 118. A 30-year-old male squash player presents to the Emergency Department following a collision with his opponent. He complains of severe chest pain and further investigation reveals three fractured ribs. There is no evidence of significant intrathoracic injury and you are satisfied that his respiratory function has not been compromised. What is the most appropriate management?

Chest strapping« YOUR ANSWER

Intercostal drain

Observation in thoracic surgery HDU

Patient Controlled Analgesia and/or intercostal block« CORRECT ANSWER

Rib belt. This patient has simple rib fractures which do not require surgical intervention. Such fractures can be extremely painful and may necessitate PCA or intercostals block analgesia or both. Once significant thoracic injury has been excluded it is safe to manage such patients on the ward or at home if their pain is controlled with oral analgesia. Rib belts and chest strapping are contraindicated. 119. A young man is about to undergo bronchoscopy for suspected inhaled peanut. Where is the most likely site for the inhaled peanut? Single best answer question – choose ONE true option only.

Right middle lobe bronchus« YOUR ANSWER

Left inferior lobe bronchus

Right superior lobe bronchus

Left superior lobe bronchus

Right lower lobe bronchus« CORRECT ANSWER. The right main bronchus is wider, shorter and runs more vertically than the left main bronchus. Consequently, foreign bodies small enough to be inhaled more commonly enter the right lung. As a result of gravity, the right lower lobe is more likely to receive such foreign bodies. 120. Following a difficult left cervical rib excision a 33-year-old female is noted to have straw coloured fluid in her drain and the diagnosis of a chyle leak is made secondary to a thoracic duct injury. Which of the following best describes the vertebral level through which the thoracic duct passes through the diaphragm? Select one answer only.

T4« YOUR ANSWER

T6

T8

MyPasTest: MRCS A Online - Jan Exam 2015 02. Anatomy; Thorax(120Qs)

-----------------------------------------------------------------------------------------------------------------

------------------------------------------------------------------------------------------------------------------------------------------------------------------------- Dr Mohammed Shamsul Islam Khan, Medical Officer, Clinical Neuro-Surgery, National Institute of Neuro-Sciences and Hospital Sher-E-Bangla

Nagar, Dhaka-1207, Bangladesh. Mobile: +880 1713 455 662, E-mail: [email protected]

Page |

123

T10

T12« CORRECT ANSWER. The thoracic duct crosses the diaphragm at the aortic hiatus (T12) and ascends the superior and

posterior mediastinum between the descending thoracic aorta (to its left) and the azygos vein (to its

right). The thoracic duct ascends anterior to the posterior intercostal vessels and has several

valves. At the thoracic inlet, it lies to the left of the oesophagus and arches forward over the dome

of the left pleura, draining into the left brachiocephalic vein. The right bronchomediastinal trunk

drains into the right subclavian vein.

Courtesy: Dr Mohammed Shamsul Islam Khan MBBS (CMC), FCPS-II (Neuro-Surgery) Medical Officer, Clinical Neuro-Surgery National Institute of Neuro-Sciences and Hospital Sher-E-Bangla Nagar, Dhaka-1207 Bangladesh. Mobile: +880 1713 455 662 E-mail: [email protected]